You are on page 1of 89

Hi guys this file is designed to be compatible with the MJDF part 1 study group , you need to

find the answers by browsing the discussions , copy the 1st 5 words of the question in the
search of the group in which it will lead you to all the discussions with answers all the best
guys, I hope this file will be of benefit to you all:

1 )A Dentist did pulpotomy of primary molar and then after few months he did stainless
steel crown. What band he gets to claim.
A-Band 1
B-Band 2
C-Band 3
D-Band 4
E-Band 5
----------------------------------------------------------------------------------------------------
2)After doing mesio-angular impaction of lower third molar, which suture material is
best used to close mucoperiosteal flap
A-3-0 Silk cutting needle
B-3-0 Vicryl cutting needle
C-Catgut
--------------------------------------------------------------------------------------------------
3 )Patient has TIA (Transient ischemic attack three months ago, you have planned
extraction and patient is on aspirin. What is the best way to proceed?
A-Delay the extraction for three months
B-Go ahead with extraction, following appropriate local measures
C-Refer to Oral surgery
D-Stop aspirin
----------------------------------------------------------------------------------------------------

4 )Patient has an over jet of above 9mm which UDA band it will be
A-Band 1
B-Band 2
C-Band 3
D-Band 4
E-Band 5
-----------------------------------------------------------------------------------------------------
5) Over jet of 4.5 mm criteria which UDA band it will be
A-Band 1
B-Band 2
C-Band 3
D-Band 4
E-Band 5
------------------------------------------------------------------------------------------------------
6) Autoclave temperatures (at what temperature the sterilization is achieved)
A-121 degrees centigrade for 15 minutes
B-121 degrees centigrade for 3 minutes
C-131 degrees centigrade for 15 minutes
-----------------------------------------------------------------------------------------------------
7) A patient with irritable bowel syndrome what do you see.
A-Mucosal blistering
B-Mucosal tags
C-Herpitiform ulcers
------------------------------------------------------------------------------------------------------
8 )Patient had pain in the chest after prolonged dental session, with an extraction
procedure, what is the best medicine to give
A-GTN
B-ASPIRIN
C- Adrenaline
D-Glucagon
-------------------------------------------------------------------------------------------------------
9) Patient is insulin dependent diabetic, complains of faint after prolonged dental session,
what is best to give
A-Oral Glucose
B-IV Glucagon
C-Insulin
D-1M Glucagon
------------------------------------------------------------------------------------------------------
10) Patient is waiting in waiting room and collapse and faints. Patient is cold, clammy, but
pulse is good. What do you give?
A-IM Glucagon
B-Oral Glucose
C-GTN
D-Hydrocortisone
E-Diazepam

11 )Patient is suffering myocardial infarction in dental chair, which position is best for
patient
A-Upright sitting position
B-Laying flat
C-Recovery position
D-Lay the pt flat and raising legs
---------------------------------------------------------------------------------------------------

12) A 90-year old gentleman presented to clinic who is edentulous and has dentures upper
and lower full is-years old. Denture bit uncomfortable, tooth structures little bit worn out,
freeway space 2-4mm, polished surfaces satisfactory and occlusal wear
minimal. How would you proceed?
A-Copy dentures
B-Hard reline
C-Soft reline
D-Construction of new dentures
-----------------------------------------------------------------------------------------------------

13) When do you, do not extract impacted 8's


A-Extraction of 8's recommended by orthodontist
B-Extraction of 8's recommended in patient undergoing renal transplant -
immunosuppresent
C-47years old with moderate periodontitis
D-Second permanent molar undergoing internal resorption due to third molar
-----------------------------------------------------------------------------------------------------
14) Non dental origin, No clinical or radiological findings. Patient complains of pain
maxillary sinus and ear deafness, unilateral ulcer on face and forehead
A-Herpes Simplex
B-Herpes Zoster
C-Paramyxo virus
D-Orthomyxo virus
E-Ep-stein bar virus
----------------------------------------------------------------------------------------------------
15) How do you check maxillary plane
A-Fox-bite-plane
B-OCcluding the upper and lower denture
C-Facebow
----------------------------------------------------------------------------------------------------
16 )What sort of matrix is best for restoring disto-occlusal restoration of 7?
A-Sectional matrix
B-Auto matrix
C-Tofflomeir matrix
----------------------------------------------------------------------------------------------------
17) How do you get incisal guidance for setting incisal teeth?
A-Study casts
B-Unsupported/supported labial fullness

-------------------------------------------------------------------------------------------------------
18) Burning mouth syndrome which part of the oral cavity is more painful
A- Tongue
B-Palate
C-Bucal Mucosa
-----------------------------------------------------------------------------------------------------
19 GTN in Angina
A-Relaxes heart muscle
B-Decreases blood Pressure
C-Increase positive ionotropic charge
D-Dilation of arterioles and venules
-----------------------------------------------------------------------------------------------------
20 had trauma - Nasal- midface area and has watery discharge and blood discharge from
nose
A-Orbital fracture
B-Zygomatic fracture
C-Lefort-l
D-Lefort-ll
-----------------------------------------------------------------------------------------------------
21) Patient with only mid-root fracture of front tooth, what is apropriate splinting time?
A-1 week
B-2 week
C-4 week
D-3 months

22) Patient with lower four central incisors with alveolar fracture, you splinted and
immobilized successfully, and then what is the next thing to do.
A-Soft diet and review
B-Antibiotics
C-Radiographic Examination
--------------------------------------------------------------------------------------------23 Patient had
trauma with minimal mobility of front teeth, no symptoms or pain or anything, what is the
best approach
A-Splinting one week
B-Splinting two weeks
C-Splinting 4 weeks
D-Splinting 6-8 weeks
E-Soft diet and review
----------------------------------------------------------------------------------------------
24 Patient had trauma 8 days ago, upper central incisor palatal luxation mild, not
interfering with occlusion but tooth non vital.
What do you do?
A-Pulp extirpation +CaoH2 dressing
B-Reposition
C-Reposition +Splinting
---------------------------------------------------------------------------------------------
25 Best material for using vital pulpotomy
A-Ferric sulphate
B-Formocresol
C-CaOH
---------------------------------------------------------------------------------------------
26 Patient had trauma on 14 (vital pulp), only thin buccal cusp remaining, best treatment
option
A-Onlay
B-inlay
C-Direct composite MODL
D-Extract and options for replacement
---------------------------------------------------------------------------------------------
27) 18 months of age, what is fluoride supplement for child living in 0.25 ppm fluoridated
water?
A-0.25mg
B- 0.5 mg
C-1 mg
---------------------------------------------------------------------------------------------
28) Six-year-old - Non fluoridated water, what is the fluoride supplement dose
A-0.25 mg
B-0.5 mg
C-1 mg
----------------------------------------------------------------------------------------------
29) Six-year-old child with fluoridated water of 0.6ppm, what is the fluoride supplement
dose given
A-0.25 mg
B-0.5 mg
C-1 mg
---------------------------------------------------------------------------------------------
30) 3-year old with fluoride concentration of 0.3 ppm fluoridated water, what is the
fluoride supplement dose given
A-0.25 mg
B-0.5mg
C-1mg
--------------------------------------------------------------------------------------------
31) Duraphat in high caries risk group children is applied every
A-2 months
B-4monttls
C-6 months
D-8 months
E-One-yearly

32 Reason for failure of eruption of lower permanent molar


A-Canine palatally place
B-Failure of eruption of central incisor
C-Absence of showing lateral incisor
D-Primary failure of eruption
------------------------------------------------------------------------------------
33 Failure of normal path of eruption
A-Abnormal position of crypt
B-Early loss of deciduous teeth
C-Supernumerary tooth
-----------------------------------------------------------------------------------
34 Cervical line and dark lines on Central incisors, molars and incisal edges, most possible
reason could be
A-childhood illness
B-Ameologensis
C-Dentinogenesis
D-Childhood illness
E-Hypoplasia
------------------------------------------------------------------------------------

35 An edentulous patient presents with ulcer in retro molar pad area first line of
management would be?!
A-Adjusting the denture
B-Incisional biopsy
C-Periapical xray

-----------------------------------------------------------------------------------
36 How should be the floor of dentalsurgery?
A-impervious
B-Absorbable
-------------------------------------------------------------------------------------
37 Possible infection that would occur from needle stick injury in a person who is
appropriately or properly immunized
A-Hepatitis B
B-Hepatitis C
C-Hepatitis A
D-HIV
-----------------------------------------------------------------------------------
38 Got a call from mother of a 3-year old child, who had trauma and avulsed the front tooth.
What instruction you will give to ??
A- Attend the surgery as soon as possible with tooth
B- Place tooth in milk and attend surgery as soon as possible
C- Place in the salt water and attend surgery as soon as possible
D- Place the tooth in its position back and attend surgery as soon as possible
-----------------------------------------------------------------------------------------

39) A child has grossly broken down upper and lower molars, carious tooth, which x-ray
would you consider
A-IOPA
B-Bitewings_
C-Vertical Bitewings
D-DPT
-----------------------------------------------------------------------------------------
40) A dental practitioner can be
A-Radiology adviser+physicist+operator
B-Supervisor + adviser+physicist
C-practitioner+refer+operator

--------------------------------------------------------------------------------------------------------

Little tooth preparation required 2


Can be made using the lost wax technique
Angle of tooth preparation should be almost vertical with sharp line angles
Has potentially the strongest properties of all ceramic crowns
Allows transillumination

Zirconion crowns
Dentine bonded crowns
Leucite reinforced crowns
Pressed ceramic crowns inceram
58 For retention of post which is not important
A-Luting cement
B-Diameter
C-Size
E-Type of post

--------------------------------------------------------------------------------------

59 RCT treated tooth, post placed, which is the best option to restore the tooth. Nayyar
technique used, good tooth structure 50%
A-Gold crown
B-Gold inlay
C-PJC
D-Gold Onlay

----------------------------------------------------------------------------------------
60 The movement of tongue on protrusion is affected to lateral side -which nerve is
involved. Patient recently had surgery and
radiation close to submandibular gland
Lingual
A-Glossopharyngeal
B-Hypoglossal
C- Chordatympani
-----------------------------------------------------------------------------------------
61 When patient complaints about a treatment and unhappy about the procedure and gives
a complaint, in how many days you
have to acknowledge the complaint
A-1 day
B-2 days
C-3 days
D-10 days
E-20 days
-----------------------------------------------------------------------------------------
64 Secondary impressions for edentulous patient close fit tray with hyper gag reflex. Which
material you use for taking an
impression
A-Alginate
B-Silicone rubber based material
C-Plaster of Paris
D-Zinc oxide eugenol
----------------------------------------------------------------------------------------
65 Palatally impacted upper maxillary canines. How would you know? Whether they are
impacted palatally or buccally and which of
the following x-rays are best option
A-Paralleling technique
B-Bisecting technique
C-Bitewings
D-OPG
E-Taking two x-rays, with paralleling technique
-----------------------------------------------------------------------------------------
66 Defective horizontal angulations while taking bitewings results in?
A-Overlapping of contact points
B-Dark radiograph
C-Light radiograph
---------------------------------------------------------------------------------------
67 Which of the following antifungal medication interact with warfarin and enhances its
effect if given Orally ?
A-Ketacanazole
B-Micanazole
C-Flucanazole
D-Amphotrocin B
----------------------------------------------------------------------------------------
68 A patient came after two days of extraction, complains he had pain for two days,
examination showed localized swelling, no lymphadenopathy. What is the first line of
treatment?
A-Possible dry socket -irrigation and placement of sedative medicament
B-Irrigation - curettage of socket, antibiotics and analgesics
C-Irrigation and antibiotics
D-Possible root piece or bone piece take IOPA x-ray and analgesics
------------------------------------------------------------------------------------------
70 Patient is having epileptic seizures continuously for 10 minutes in dental surgery - first
line of management is
A-Place in recovery position
B-Give diazepam
C-Oxygen
D-Intranasal or buccal midolazam
-----------------------------------------------------------------------------------------
71 Sudden unilateral onset of facial swelling and pain over parotid area, which is not yet
fluctuant
A-Bacterial saliadenitis
B-Salivary duct infection
C-Saliorrhea
D-Viral siliadenitis

---------------------------------------------------------------------------------------
72 What do you see in irritable bowel syndrome?
A-Mucosal blistering
B-Herpitiform ulcers
C-Mucosal tags
D-Macular patches on buccal mucosa
---------------------------------------------------------------------------------------
73 A fifteen year old patient asks for bleaching of teeth. What is the first line of
management?
A-Discuss options with both patient and parent
B-Discuss option with both patient and parent, once you have obtained consent from
patient
C-Discuss only with patient
D-Discuss only with parent
----------------------------------------------------------------------------------------
74 A child avulsed permanent tooth came with her aunt, what will you do?
A-Do not do anything
B-Do not do treatment as aunt is not legal guardian of the child
C-Do not do treatment as child's mother is not there to give consent
D-Re-implant and splint it with aunt's consent
------------------------------------------------------------------------------------------
75 Best ways to reduce radiation for patient who is going to have IOPA X-rays
A-Lead apron
B-Paralleling technique
C-Rectangular collimation
D-Film
------------------------------------------------------------------------------------------
78 What is the best cement used to cement minimally done inlay restoration (MOD)
A-GIC
B-ZOE
C-Zinc phosphate
D-Zinc poly carboxylate
E-Resin cement
-----------------------------------------------------------------------------------------
79 What sort of matrix is best for restoring disto-occlusal restoration of 7?
A-Sectional matrix
B-Auto matrix
------------------------------------------------------------------------------------------
80 Patient had trauma with minimal mobility of front teeth, no symptoms or pain or
anything, what is the best approach
A-Splinting one week
B-Splinting two week
C-Splinting 4 weeks
D- Soft diet and review

83-Deciduous tooth had intrusion 61, what is most likely to occur for permanent tooth
A-Impaction
B-Uneruption
C-Hypoplasia
D-Dilaceration
-----------------------------------------------------------------------------------------
84-Splinting time for Avulsion
A-2 weeks
B-1 week
C-3 weeks
---------------------------------------------------------------------------------------
85 Got a call from mother of a 3-year old child, who had trauma and avulsed the front tooth.
What instruction you will give to
mother
A-Place tooth in milk and attend surgery as soon as possible
B-Place in the salt water and attend surgery as soon as possible
C-Place the tooth in its position back and attend surgery as soon as possible
D-come with the child to the surgery as soon as possible and bring the tooth with you.
-------------------------------------------------------------------------------------
86 A 3-year-old boy avulsed 61, and mother re-implanted it in its socket and clot is holding
the primary tooth. IOPA shows
permanent tooth present, what would you do
A-Leave it and review:
B-Splinting for one week
C-Remove and space maintainence
---------------------------------------------------------------------------------------
87 A child has grossly broken down upper and lower molars, carious tooth, which x-ray
would you consider
A-IOPA
B-Bitewings
C-Vertical Bitewings
D-OPG
----------------------------------------------------------------------------------------
88 When patient complaints about a treatment and unhappy about the procedure and gives
a complaint, in how many days you
have to acknowledge the complaint
A-1 day
B-3 days
C-10 days
D-20 days
---------------------------------------------------------------------------------------
94 Which of the following antifungal medication interact with warfarin and enhances its
effect IF used topically?
A-Ketacanazole
B-Micanazole
C-Amphotrocin B
D- Nystatin
----------------------------------------------------------------------------------------
95 Patient has cardiac arrhythmias on warfarin and yesterday had INR of 3, today needs
extraction. What do you do?
A-Differ the extractions
B-Alter the dose of warfarin
C-Do the extraction and control post operative bleeding
-------------------------------------------------------------------------------------------
106 Which test is the most reliable test to indicate the presence of active hepatitis
A-H bs(su rface }antigen
B-Hbc(core)antigen
C-Hbe antigen
D-Antibody to HBs antigen
------------------------------------------------------------------------------------------
107 First line treatment to needle stick injury
A-Wash the area under running water
B-Scrub the area
C-Refer to infectious disease specialist
D-Call emergency
---------------------------------------------------------------------------------------
108 what is legal amount (unit}of alcohol intake for men
A-18
B-21
C-24
-------------------------------------------------------------------------------------
109 Medical condition of pt who reserved a seat in a dinner meating for his dead wife
A-Mania
B-Anxiety
C-Depression
D-Schizphrenia
----------------------------------------------------------------------------------------
110 Lady in 40s got TMJpain for about few months, she got divorced recently what is the
first line treatment
A.amitrptyline
B.physiotherapy
C.occlusal splint
--------------------------------------------------------------------------------------------
111-what type of cells that proliferate in later stage of her life?
A.Odontoblast
B.cemtoblast
C. rest of malazess
E.undifferentiated mesynchymal cells
--------------------------------------------------------------------------------------------
112 BLS no of compressions
A-40:1
B-30:1
C-30:2
-------------------------------------------------------------------------------------------
113 Patient fit and healthy then he got cold and clammy then he lost his consciousness
A-Syncope
B-Epilepsy
C-Hypoglycaemia
-----------------------------------------------------------------------------------------
115 The term abrasion best describes:
A.Loss of substance by chemical agent
B.Loss of substance by external agent
C.Loss of substance by the movement of tooth against tooth
D.The rapid loss of substance that is seen in the movement of porcelain crowns aganist
natural teeth
-----------------------------------------------------------------------------
116 Some days after preparation and filling of a shallow class I amalgam cavity the patient
complains of pain on biting. You would:
A.Perform vitality test
B.Replace filling
C.Check for premature contacts
D.Remove all occlusal contacts from this filling
E.Tell the patient to wait 2-4 weeks, the pain will go away
--------------------------------------------------------------------------------------
117 In a composite filling, the matrix band is for
A.Help shaping and contouring the filling
B.Prevent material to be pushed under the gingival margin
-----------------------------------------------------------------------------------------
120 In a class II div 2 malocclusion, which bridge design would be contraindicated for a
missing lateral upper incisor?
A.Cantilever bridge
B.Maryland bridge
---------------------------------------------------------------------------------
121 Which is the best cantilever bridge design for missing maxillary canine? Abutment on
A.Both premolars
B.Lateral and central incisor
C.Lateral incisor
D.First premolar
---------------------------------------------------------------------------------
122 A 9 years-old child who has sustained a fracture of a maxillary permanent central
incisor in which 2 mm of the pulp is exposed, presents for treatment 30 minutes after
injury. Which of the following should be considered?
A.Remove 1-2 mm of the pulp tissue surface, place calcium hydroxide and fill with resin
B.Remove 1-2 mm of the pulp tissue surface and cover with ledermix
C.Place calcium hydroxide directly on the exposed pulp
D.Pulpotomy using formocresol
E.Pulpectomy and immediate root filling

124 The most important dietary habit for caries development is


A.Amount of sugar intake
B.Frequency of sugar intake
C.Form of sugar intake
-----------------------------------------------------------------------------------------
125 The normal un stimulated salivary flow rate is
A.0.02 ml/min
B.0.2 ml/min
C. 2 ml/min
-------------------------------------------------------------------------------------------
126 Titanium is used in dentistry
A.ln a very pure form in implants
B.ln an alloy with aluminium in casting for crowns and bridges
C.ln an alloy with nickel in orthodontic wires
D.A and B
E.A, B and C
-------------------------------------------------------------------------------------------
128 How is the regeneration process after damage by injury to odontoblasts working?
A.Proliferation of the remaining odontoblasts
B.Differentiation from fibroblasts
C.Regeneration from undifferentiated mesenchyme I cells
D.Histodifferentiation from ectodermal cells
E.Differentiation from the inflammation cells
-------------------------------------------------------------------------------------------
130 Reversible pulpitis is characterized by
A.Pain lasts longer on hot or cold stimulus than normally
B.Patient can't localize pain
C.Wili have periapical involvement in radiograph
------------------------------------------------------------------------------------------
131 Irreversible pulpitis is characterized by
A.There is often a history of spontanous pain
B.Sudden throbbing pain
C.Pain can't be localised when it reaches the peri apical area
D.There is pain which lingers for a short duration after removal of stimulus
---------------------------------------------------------------------------------------
132 What are the symptoms of internal resorption?
A.very painful
B.Symptom-free or only mild pain
------------------------------------------------------------------------------------------
133 What kind of root fracture in a tooth has the best prognosis? A fracture at the
A.Apical third
B.Coronal third
C.Middle third
D.Vertical fracture
-------------------------------------------------------------------------------------------
135 which of these disease that caused by positive lacto bacillus
A-Syphilis
B-TB
C-Angular cheilitis
D-Dental Caries
---------------------------------------------------------------------------------------------
136 Child with sore throat and feeling un well, he got macular rash on his cheek?
A.measles
B.chicken pox
C.herpes simplex
---------------------------------------------------------------------------------------------
137 Diabetic pt with abscess complain of failure of root canal therapy and during the
examination the filling was leaking? What the
reason
a.lack of coronal seal
b.lack of apical seal
c. Pt medical condition
------------------------------------------------------------------------------------------
138 what is the most common site that affected by burning mouth syndrome?
A-Tongue
B-Palate
C-Cheek
-----------------------------------------------------------------------------------------
140 Pt got recently metallic taste sensation after taking oral medication?
a.burning mouth syndrome
b.oral dysthesia
c.oral dysgeusia
----------------------------------------------------------------------------------------
141 Reasons for crowding
A early loss of deciduous teeth
b.delayed eruption of lower first molar
c primary failure of eruption
-----------------------------------------------------------------------------------------

143 Reason for delayed eruption of lower first molar


a ea rly loss of decid uous teeth
b.delayed eruption of lower first molar
c primary failure of eruption
-------------------------------------------------------------------------------------------
142 Reason for palatal position of upper canine
a early loss of deciduous teeth
b.delayed eruption of lower first molar
c primary failure of eruption
d abnormal position of crypt

--------------------------------------------------------------------------------------------
148) A Gracey curette is characterized by
A-The blade and the shank form a 902angle
B-Can be used on both sides
C-Can be used on any tooth surface
D-It is specific for each surface ofthe tooth
---------------------------------------------------------------------------------------------
149 A patient with no positive history came along for scaling. The moment you pick up
your anterior scaler you punch your finger, what should you do?
A-Complete the procedure as if nothing has happened
B-Check dentist's blood for Hepatitis B antibody HBsAb
C-Check dentist's blood for Hepatitis B antigen HBsAg
D-Check patient's blood for Hepatitis B antibody HBsAb and HIV antigen HIVAg
E-Dentist should go and take a HBsAb vaccine
--------------------------------------------------------------------------------------------
150 A patient on the dental chair has cardiac arrest. What is INCORRECT,
A-Observing the vital signs and check that the air way is clear is at high importance
B-Expired air has 15% 02 only, and cardiac compressions achieve 30-40% of cardiac
output
C-Intermittent positive pressure at the rate of 40/min will reduce the chances of erebral
hypoxia
D-Intermittent positive pressure is better than mouth to mouth when it has been given at
the same rate.
E-You check the pulse and respiration of the patient before starting any resuscitation
-------------------------------------------------------------------------------------------
151 Treatment for small carious cavity in co operative child
A-Pulp capping
B-Pulpotomy
C-Pulpectomy
D-simple restoration
E-extraction
-------------------------------------------------------------------------------------------
152 Treatment for pinpoint exposure in symptomless tooth
A-Pulp capping
B-Pulpotomy
C-Pulpectomy
D-Extractiom
--------------------------------------------------------------------------------------------
154 Pt with labially impacted upper canine, how does u know clinically?
A-Swelling on the labial area
B-Swelling on the palatal side
--------------------------------------------------------------------------------------------
155 First line management of dry socket
a. irrigation.x ray metrondiazole 200
b. irrigation xray metro diazole 400
c. irrigation and x ray only
--------------------------------------------------------------------------------------------
156 First line management of pt with palatal dietary erosion
a. crown.,
b. palatal veener
c. direct composite
d.indirect composite
-------------------------------------------------------------------------------------------
166 Pt on warfarin, what is the accepted inr to do simple extraction
A-2
B-3
C-4
D-5
------------------------------------------------------------------------------------------
169 what the feature of the carpet of surgery?

------------------------------------------------------------------------------------------
72 Child accidentally consumed 5mg/kg flouridewhat the first line treatment?
A-Give child salty drink
B-Give child sugary drink
C-Give child milk
-----------------------------------------------------------------------------------------
173 What is the most correct flap design in apecictomy?
A-Apical re position
B-Apex is broader tha n the base
C-Base is broader than the apex
D-Apex and base are equal

-----------------------------------------------------------------------------------------
174 Common irrigation solution for endodontices?
A- sodium hypochlorite
B- Saline
C- Chlorhexidine
----------------------------------------------------------------------------------------
175 What will develop after prophylaxis?
A.Aceliular pellicle is formed immediately after
B.Celiular pellicle is formed immediately after
C.Acellelar pellicle is formed after 48 hours
D.Cellular pellicle is formed after 48 hours
-----------------------------------------------------------------------------------------
176 What is the purpose of making a record of protrusive relation and what function does
it serve after it is made?
A-To register the condylar path and to adjust the inclination of the incisal guidance.
B-To aid in determining the freeway space and to adjust the inclination of the incisal
guidance.
C-To register the condylar path and to adjust the condylar guides of the articulator so that
they are equivalent to the condylar paths of the patient.
D- To aid in establishing the occlusal vertical dimension and to adjust the condylar guides
of the articulator so that they are
equivalent to the condylar paths of the patient.
-----------------------------------------------------------------------------------------
177 A 50 years-old patient presents with pain from time to time on light cervical abrasions.
What is your first management to help patient in preventing pain in the future?
A.Change diatary habits
B.Change brushing habits
C.GIC fillings
-----------------------------------------------------------------------------------------
178 In planning and construction of a cast metal partial denture the study cast
A-facilitates the construction of custom trays
B-minimizes the need for articulating
C-provides only limited information about inter ridge distance, which is best assessed
clinically
D-can be used as a working cast when duplicating facilities are not available

--------------------------------------------------------------------------------------------------------

179 What are the artificial teeth in removable dentures made of?
A.Porcelain
B.Cross-linked methyl-methacrylate
C.Ethyl-methacrylate
D.Acrylic
---------------------------------------------------------------------------------------
180 Following extraction of the molar teeth
A-The ridge height is lost more from the maxilla than from the mandible
B-The maxillary ridge will get more bone lost from the palatal aspect than the buccal
C-The mandibular arch is relatively narrower than the maxillary arch
D-Compared with the pre-resorption state, the mandibular ridge will lose more bone from
the lingual aspect than the buccal one.
------------------------------------------------------------------------------------------
181 Which anatomical landmark is important to include in impressions for lower full
dentures?
a. Mylohyoid ridge
B.Lower incisive papilla
------------------------------------------------------------------------------------------
182 Wrought metal is to be,
A-Marble
B-Quenched
C-Has undergone cold treatment during processing
---------------------------------------------------------------------------------------------
183 Which of the following is ONE indication for indirect pulp capping?
A.When further excavation would lead to pulp exposure
B.Excavation of a very deep caries
---------------------------------------------------------------------------------------------
185 A major difference between light cured and chemical cured composite is that during
setting or in function the light cured
materials tend to:
A-Seal the margins better and completely
B-Exhibit less wear on time
C-Undergo greater colour change
D-Shrink more rapidly
E-Posses greater fracture toughness
---------------------------------------------------------------------------------------------
186 What consideration is important in deciding if a bridge for upper missing incisors
should be made in pontic design or with gingiva imitation?
A.Wishes of patient
B.Bone resorption in edentoulos span
--------------------------------------------------------------------------------------------

187 The most common cause of porosity in porcelain jacket crowns is,'
A-Moisture contamination
B-Excessive firing temperature
C-Failure to anneal the platinum matrix
D-Excessive condensation of the porcelain
E-Inadequate condensation of the porcelain
---------------------------------------------------------------------------------------------
188 How should the occlusion in partial removable dentures be designed?
A. Artificial teeth should be out of occlusion
B. Artificial teeth should not interfere with the incisal guidance established by the remainig
natural teeth.
-------------------------------------------------------------------------------------------
189 The minimal labial tooth reduction for satisfactory aesthetics with porcelain fused to
metal crown is,
A.1mm
B-The full thickness of enamel
C-1.5 mm
D.2.5mm
E. One third of the dentine thickness
----------------------------------------------------------------------------------------------
190 In removable partial denture, the principle of an indirect retainer is to:
A-Stabilize against lateral movement
B-Prevent settling of major connectors
C-Restrict tissue movement at the distal extension base of the partial denture
D-Minimize movement of the base away from the supporting tissue
-------------------------------------------------------------------------------------------
191 When a removable partial denture is terminally seated the retentive clasps tips should:
A. Apply retentive force into the body of the teeth
B.Exert no force
C.Be invisible
D.Resist torque through the long axis of the teeth
-------------------------------------------------------------------------------------------
192 Glass lonomer Cement sets because of,
A-Acid-Base reaction
B-Addition polymerisation reaction
C-Growth of glass crystals
D-Slip plane locking
E-Solvent evaporation
-------------------------------------------------------------------------------------------
193 The reflex in gagging patients is caused by:
A-Trigeminal nerve
B-Glossopharyngeal
C-Facial nerve
D-Recurrent laryngeal
--------------------------------------------------------------------------------------------
194 The use of nickel chromium in base plate should be judiciously considered because:
A-A significant number of females are allergic to nickel
B-A significant number of females are allergic to chromium
C-A significant number of males are allergic to nickel
D- A significant number of males are allergic to chromium
-----------------------------------------------------------------------------------------
195 Which of the following liquids is not suitable for prolonged immersion of cobalt
chrome partial dentures:
A. Alkaline peroxidase
B. Sodium hypochlorite
C.Soap solutions
D.Water
----------------------------------------------------------------------------------------------
196 In complete dentures, cheek biting is most likely a result of:
A-Reduced Overjet of posterior teeth
B-To high vertical dimension
C-Teeth have large cusp inclines
---------------------------------------------------------------------------------------------
197 The most common cause of fracture at the isthmus of a class II dental amalgam
restoration is:
A.lnsufficient condesation
B.Fracture line developing from pulpal-axial angle of the cavity
C.Underconturing of the isthmus area
D.Moisture contamination of the amalgam during placement
E.lnadequate bulk of amalgam at pulpo-axialline angle
--------------------------------------------------------------------------------------------
198 Why do you overpack amalgam fillings?
A. To remove excess mercury
B To prevent microleakage

---------------------------------------------------------------------------------------------

199 What is CORRECT in regard to the periodontal surface area in maxillary teeth:
A-central incisor> first premolar> second premolar
B-Canine> first premolar> central incisor
C-Canine> lateral incisor> second premolar
D-Canine> central incisor> first premolar
-----------------------------------------------------------------------------------------
200 When restoring with composite resins, why do we do the cavo-surface bevelling:
A.Aesthetic
B.To open enamel rods for acid attack
C.To smooth preparation
D.A and B
E.AII of the above
------------------------------------------------------------------------------------------
201 A well constructed complete denture:
A-Needs little maintenance
B-Needs less than a week for adjustment and total success
C-Has adverse effects and decreases taste sensations
-----------------------------------------------------------------------------------------
202 On examination of a composite restoration you find a dark Stain:
A. Replace the composite
b. Repair with unfilled resin
c. Apply topical fluoride at the margin
--------------------------------------------------------------------------------------
203 A patient complains of sensitivity, on examination you find a composite filling restoring a good
cavity preparation without any secondary caries; what is your next step:
A-Extirpate the pulp that is obviously inflamed
B-Place ZOE dressing to sedate the pulp
C-Ask patient to come back in six months
D-Repeat restoration
--------------------------------------------------------------------------------------
204 What is the best way to cement a Maryland bridge,
A-GIC
B-Resin cement
C-Composite resin
D-Zinc Phosphate cement
E-Oxide Zinc and eugenol
----------------------------------------------------------------------------------------
205 The ideal length of a post in the fabrication of crown and core of endodontically treated tooth is:
A.2/3 of tooth length
B.1.5 times that of the crown
C.Yz root length
D.The length of the crown
-----------------------------------------------------------------------------------------

206 While you finish a class I cavity, the enamel is sound but you notice a thin brown line ln
the dentine and on the dentino-enamel
junction, what is your response,
A-You leave it and complete the final restoration
B-You extend your preparation and clean it
C-You apply a cover of varnish
------------------------------------------------------------------------------------------
207 Dental caries ofthe proximal surfaces usually starts at,
Somewhere between the ridge and the contact area
A-Just gingival to contact areas
B-Just above the gingival margin
C-At the contact point
--------------------------------------------------------------------------------------------
208 The Frankfort plane is defined by which anatomical landmarks,
A-Porion, orbitale
B-Sella, orbitale
C-Nasion, Tragus
-------------------------------------------------------------------------------------------
211 The advantage of the silicone in soft relining material over hard plastic acrylic
materials is,
A-Capability to flow
B-Prevents the colonization of Candida albicans
C-Resilient in long run
D-Better bond strength
-------------------------------------------------------------------------------------------
212 A female patient comes to you complaining of persistent pain in a heavily restored
central incisor; you suspect irreversible pulpitis and you have been told that she is in
transit leaving by plane next day. Your treatment will be,
A-Remove filling and place a sedative dressing
B-Pulpectomy and Ledermix dressing
C-Pulpectomy and calcium hydroxide dressing
D-Prescribe analgesics and systemic antibiotic
------------------------------------------------------------------------------------------
214 Following calcium hydroxide pulpotomy, the dentist would expect dentine bridge to
form at,
A-The exact level of amputation
B-Level somewhere below the amputation
C-Half way between amputation and apex
D-At the apical region of the tooth
--------------------------------------------------------------------------------------

215 In the construction of a full veneer gold crown, future recession of gingival tissue can
be prevented or at least minimized by,
A-Extension of the crown 1 mm under the gingival crevice
B-Reproduction of normal tooth incline in the gingival one third of the crown
C-Slight over contouring of the tooth in the gingival one fifth of the crown
D-Slight under contouring of the tooth in the gingival one fifth of the crown
--------------------------------------------------------------------------------------
216 What is correct in regard to high copper amalgam,
A-Reacts and strengthens the amalgam by its dispersion properties
B-Reacts to form copper-tin phase thereby eliminating the tin-mercury phase
C-Reacts to form copper-silver phase there by eliminating the silver mercury phase
D-Reacts and strengthens the amalgam by its grain diffusion
--------------------------------------------------------------------------------------------
217 The removable partial denture requires relining,what would be the most appropriate
action,
A-take an impression by asking the patient to occlude on it
B-Provide equal space between denture and gingival tissues.
C-Make sure the framework and retainers are seated in place before taking impression
----------------------------------------------------------------------------------------------
218 Stiffness of material is measured by
A-Proportional unit
B-Modulus of elasticity
C-Stress/ stra i n
D-Ultimate tensile strength
--------------------------------------------------------------------------------------------
219 Two central incisors on a radiograph are showing with what looks like eye drop
radiolucency. You decided to start endodontic treatment on these teeth but when you tried
to open access to the root canal you find clearly closed orifices with what look like
secondary dentine. What is your initial management?
A-Leave as it is and start a permanent restoration.
B-Start systemic antibiotic
C-Try to ream and file canals
-------------------------------------------------------------------------------------------
220 After the initial development stage and in the absence of pathology, the size of the pulp
chamber has been reduced by,
A-Deposition of primary dentine
B-Deposition of secondary dentine
C-Reparative dentine
D-Pulp fibrosis
------------------------------------------------------------------------------------------
223 Denture stomatitis is commonly associated with,
A-The continuous wearing of removable orthodontic appliances in otherwise healthy
patient
B-The proliferation of hypertrophic tissue at the denture periphery
C-The overgrowth of some constituents of oral normal microflora
D-Allergy to denture base material
------------------------------------------------------------------------------------------
224 The light emitted by the polymerization lamp has to be checked from time to time. The
meter used for this only measures light
in the range of:
A-100-199 nm
B-200-299 nm
C-300-399 nm
D-400-499 nm

225 Which is correct in regard to shade selection of crowns:


A. It should be selected before starting preparation
B.Chroma is the lightness/darkness of colours
C.Value is the colour itself
D.Hue is the concentration of colours
------------------------------------------------------------------------------------------
226 Where would you expect to find the mylohyoid muscle in relation to the periphery of a
full lower denture:
A.Mandibular buccal in the midline
B.Mandibular lingual in the first premolar area
C.Mandibular lingual in the midline
D.Mandibular disto buccal area
-----------------------------------------------------------------------------------------
227 After reimplantation of an avulsed tooth the prognosis may be poor because of
A.External resorption
B.lnternal resorption
-----------------------------------------------------------------------------------------
228 .2 mg of NaF contains how many mg of fluoride ions?
A.O.5mg
B.1.0 mg
C.1.5mg
D.10mg
E.20 mg
-----------------------------------------------------------------------------------------
229 How would you treat denture stomatitis?
A.Nystatin
B.Tell the patient to leave the denture out for some days
------------------------------------------------------------------------------------------
230 What is true about third molar surgery?
A.Swelling is maximum after 24 - 48 hours
B.Mental paraesthesia indicates careless technique
------------------------------------------------------------------------------------------
2311n anaesthizing a 70 kg healthy man with Lignocaine 2% with 1:100,000
vasoconstnctor.what is correct?
A.The toxic threshold is 22 ml
B.2.2 ml is the maximum you can give in one session
C.Lignocaine has the same anaesthizing capacity as Benzocain
D.Lignoscain is 5 times less potent than Bupivacain
E.Lignocain is more dangerous in Hypothyreodism than Bupivacain
----------------------------------------------------------------------------------------
232 A suddenly swollen upper lip that lasts for 48 hours or more is most likely
A.Haemangioma
B.Agioneurotic oedema
C.Mucocele
D.Cyst
---------------------------------------------------------------------------------------
233 What is the most important factor to reduce radiation in dental radiographs?
A-Speed of film
B-Collimation
C-Filtration
D-Cone shape and length
E-Use of lead apron
-----------------------------------------------------------------------------------------
234 What is the best way for a permanent decline in caries of a population?
A.Change diatary habits
B.Topic and water fluoridation
C.Awareness of dental health matters
D.Better tooth brushing
E.Nurse advice
---------------------------------------------------------------------------------------
235 Loss of sensation/paraesthesia in the lower lip may be produced by,
A-Bell's palsy
B-Traumatic bone cyst
C-Trigeminal neuralgia
D-Osteomyelitis
E-Ludwig's angina
--------------------------------------------------------------------------------------
236 In anxoius and psychologically stressed patients gingivitis is often more severe
because of
A.Stress causes histamine and serotonine release
B.5tress causes catecholamine and corticosteroid release
c.Stressed people neglect their oral hygiene
---------------------------------------------------------------------------------------
237 A retained lower primary incisor causes the permanent incisor to
A.erupt buccally
B.erupt lingually
C.ankylose
-----------------------------------------------------------------------------------------
238 What is the least probable consequence in thumb-sucking?
A.Reclining of lower incisors
B.Protrusion of upper incisors
C.Formation of deep palate with big overbite
-------------------------------------------------------------------------------------------
239 Ankylosis of teeth is often found after changes in the continuity of the occlusal plane.
These changes are caused by
A.Overeruption of opposing teeth
B.Localised growth inhibition of the alveolar bone
C.Sinking of ankylised tooth into the bone
------------------------------------------------------------------------------------------
240 What is not important in obturation materials for primary teeth?
A.Good apical seaL
B.Radioopacity
C.Antibacterial
D.Resorbable
------------------------------------------------------------------------------------------
241 What is your first consideration in the treatment of dry socket?
A.Prevention of osteomyelitis
B.Pain relief
-----------------------------------------------------------------------------------------
242 A patient presents with pain in the upper left segment. On inspection you f~nd a
localized alveolar abcess distal 27. What will be you management? -
A.Drainage
B.Extraction of tooth
-----------------------------------------------------------------------------------------
243 What does not help in establishing the caries risk in children?
A.History of caries
B.Lactobacilius count
C.Dietary habits
D.Brushing habits
E.Genetic predisposition
-----------------------------------------------------------------------------------------
244 Opioid drugs are similar to which endogenous substances?
A-Bradykinins
B-Peptides
C-Prostaglandins
D-Serotonins
E-Enkephalins
-------------------------------------------------------------------------------------------
245 Which of the following have a tendency to recur if not treated correctly?
A-Giant cell granuloma
B-Lipoma
C-Fibrous epulis
D-Haematoma
E-Pulp polyps
------------------------------------------------------------------------------------------
246 When no radiation shield is available, the operator should stand out of the primary x-
ray beam at a distance from the patient's
head of at LEAST:
A-0.5 metres
B-1 metre
C-1.5 metres
D-2 metres
E- 3 metres
--------------------------------------------------------------------------------------------
247 Which of the following is a typical consequence of dental crowding, assuming no
primary tooth has been lost prematurely?
A-Overlapping of lower incisors
B-Palatal displacement of upper canines
C-Impaction of 15 and 25 between first premolars and first molars
D-Mesial tipping of 16 and 26
E-Rotation of 16 and 26
----------------------------------------------------------------------------------------
248 What is the dominant microflora in acute necrotic ulcerative gingivitis (ANUG)?
A-Spirochaetes and Fusobacterium SP
B-Spirochaetes and eikenella corrodens
C-Polymorphs and lymphocytes
D-Actinobacillus actinomycetes comitans oral capnocytophaga
E-Porphyromonas gingivalis and prevotella intermedia
-------------------------------------------------------------------------------------------
249 Which of the following is true regarding gingivosis (Desquamative gingivitis)
A-It is caused by hormonal imbalance
B-Is seen only at or after menopause
C-Is frequently caused by lichen planus
D-Is a variant pregnancy gingivitis
E-Is related to nutritional disturbance
----------------------------------------------------------------------------------------
250 What are the points that determine the facial line in cephalometric points
A-Nasion, pronasale, pogonion.
B-Sella, nasion, pogonion
--------------------------------------------------------------------------------------------
251 A 10 year old boy presents with small greyish white lesion surrounded by a red halos
on the soft palate and tonsillar pillars, small vesicles are found. He has fever and pain in the
ear and won't eat. The MOST probable diagnosis is?
A.Herpangina
B.Measles
C.Primary herpetic stomatitis
-----------------------------------------------------------------------------------------
252 A 12 years-old child presents with symptoms of widespread gingivitis with bleeding
and general malaise for several weeks. How
would you manage this patient?
A-Prescribe Metronidazole 100mg
B-Locally debride, give oral hygiene instruction and prescribe H202 mouth wash.
C-Give a prophylaxis with ultrasonic scaling
D-Refer for hematological screening
E-Advice for bed rest with supportive and palliative treatment
-------------------------------------------------------------------------------------------

253 What is the effect of office dental prophylaxis of regular six month intervals on
children's oral health?
A-Reduced caries incidence by approximately 30%
B-Provide a long term improvement in oral hygiene
C-Provide a short term improvement in oral hygiene
D-Prevent gingivitis
E-Reduce the need for patient cooperation
-----------------------------------------------------------------------------------------
254 What is the most frequent cause of pain which occurs several days after obturation?
A-Entrapped Bacteria in the periapical region
B-Underfilling the root canal system
C-Overfilled root canal
-------------------------------------------------------------------------------------------
255 A diabetic patient with moist skin, moist mouth and weak pulse; what would you do:
A-Give glucose orally
B-Administer 02
C-Administer adrenaline
D-Inject insulin
------------------------------------------------------------------------------------------
256 A patient has developed a sever chest pain and difficulties in breathing while in the
dental chair. Your initial response is:
A-Administer glycerine trinitrate and monitor patient in upright position
B-Patient has an acute episode of angina as demonstrated by curve in ECG
C-No treatment is required until confirmed as MI by ECG
D-Patient has myocardial infarction as confirmed by ECG
-----------------------------------------------------------------------------------------
257 In the case of malignant melanoma occurring intraorally, which of the following is true:
A-Uncommon on the palate
B-Should not be biopsied, as this will increase metastasis
C-The 5 years survival rate is 20%
D-The incidence of oral melanoma is the same as those on the skin

258 What is NOT TRUE in relation to the use of diazepam for sedation?
A-Patient commonly complain of postoperative headache
B-An acceptable level of anxiolytic action is obtained when the drug is given one hour
preoperatively
C-There is a profound amnesic action and no side effects
D-Active metabolites can give a level of sedation up to 8 hours post operatively
C-Can be used safely for children
-----------------------------------------------------------------------------------------
259 Patient on anti-coagulant therapy requires an extraction to be performed. Which of the
following is NOT true:
A-Minor leedings bleeding can be reduced somehow by using tranexamic acid
B-Prothrombin value above 2.5 is required to perform extraction
C-It takes up to 12 hours for Vitamin K reverse effects of warfarin
D-Heparin can be administered sub-cutaneous and acts rapidly
-----------------------------------------------------------------------------------------
260 A physician refers a nine year old boy to you to confirm diagnosis. The boy has a fever
of 40C and coughing. When you focus your light into his eyes he turns away. Intra-orally
there are white spots surrounded by red margins. The disease and lesions are:
A-Measles and the spots are Koplik's spots
B-AHGS vesicles
C-Rubella and the spots are Fordyce's spots
----------------------------------------------------------------------------------------
261 What is true in TMJ dysfunction therapy?
A.Should be treated surgically
B.Appliances that raise the bite usually relief the symptoms and are used prior to any
surgery
------------------------------------------------------------------------------------
262 What is true regarding pregnancy gingivitis?
A.lt is due to increased gingival microcirculation
B.Elevated estrogen and gestagen levels are directly responsible .;
C.Hormonal changes cause the growth of anaerobic bacteria (Prevotella intermedia)
------------------------------------------------------------------------------------
263 5 mm probing depth means:
A.Patient has periodontitis
B.Probe is 5 mm beyond gingival margin
C.Probe is 5 mm beyond dentino-enamel junction
-------------------------------------------------------------------------------------
264 A middle aged woman gives a history of intermittent unilateral pain in the sub
mandibular region, most probable cause is,
A-Calculus in the salivary duct resulting in sialolithiasis.
B-Ranula
C-Cyst
D-Mucocele
---------------------------------------------------------------------------------------
265 By which of the following mechanism reduces Aspirin pain:
A-It is anti inflammatory by the release of histamine
B-It blocks the cyclo-oxygenase pathway.
-------------------------------------------------------------------------------------
266 In minor oral surgery, what is TRUE in regard to antibiotics:
A-Amoxil satisfactorily covers the dental spectrum
B-Metronidazole and Amoxil have the same penetrating power
C-It is evident that it will reduce post operative swelling
D.There is convincing evidence that Prophylactic prescription of antibiotics will reduce
postoperative infections
E.Most oral infections get anaerobic after 2 to 3 days
---------------------------------------------------------------------------------------
267 A patient comes with a firm, painless swelling of lower lobe of parotid which has
grown progressively for the past year. He complains of paresthesia for the past 2 weeks.
This is most likely to be:
A-Pleomorphic adenoma
B-Carcinoma of the parotid
C-Lymphoma of parotid
-----------------------------------------------------------------------------------------
268 What is true in treating a patient with secondary herpes simplex:
A-Acyclovir inhibits viral transcription when applied in the prodromal phase
B-Idoxuridine is better than acyclovir when applied topically
C-Antivirals are contraindicated in immuno-compromised patient
----------------------------------------------------------------------------------------
269 During extraction of a maxillary third molar the tuberosity is fractured; however, it
remains in place attached to the
mucoperiosteum. Which of the following procedures should be employed:
A-Remove the tuberosity and suture
B-Leave the tuberosity and stabilize if required
C-Remove the tuberosity and fill the defect with Gelfoam then suture.
D-If fractured tuberosity is greater than 2 ern, leave in place and suture
----------------------------------------------------------------------------------------------
270 An incision biopsy of an ulcerated and intruded clinically suspicious lesion in a SO year
old female reveals chronic inflammation;
you would:
A-Inform the patient and her physician of your findings and instruct the patient to return in
six months
B-Surgically excise the entire lesion since you know it is not malignant
C-Dismiss the patient with instructions for warm saline rinses and re-examination
D-Repeat the biopsy
----------------------------------------------------------------------------------------
271 Reducing the size of the focal spot will:
A-increase sharpness
B-increase density
------------------------------------------------------------------------------------------
272 The initial priority in treatment of horizontal fractures is:
Preservation of pulp
A-Immobilisation
B-Root canal treatment
C-Calcium hydroxide treatment
-----------------------------------------------------------------------------------------
273 Which of the following has proven to be the MOST important in community preventive
program:
A-Dental awareness of the community
B-Institution of oral hygiene measures
C-Water fluoridation
---------------------------------------------------------------------------------------
274 What effect has placing a sealant over pits and fissures on the progression of caries?
A.Decreased new caries
B.lncreased new caries
C.Progression of existing caries
D. No effect on existing caries
-----------------------------------------------------------------------------------
275 In advanced periodontitis with marked mobility, teeth may be splinted:
A.To improve comfort for the patient
B.splinting helps in transmitting the force to the adjucent teeth to reduce the load on the
involved teeth
-----------------------------------------------------------------------------------
276 Swallowing will aid in the diagnosis of:
A-Branchial cyst
B-Thyroglossal duct cyst
C-Ranula
D-Retention cyst
E. Glossothyroid cyst
--------------------------------------------------------------------------------------
277 Which of the following will increase sharpness:
A-Larger focal spot
B-Smaller focal spot
C-Increase object-film distance
D-Decrease patient-source distance
-----------------------------------------------------------------------------------------
278 In severe periodontitis the probe will eventually be:
A-prevented to go deeper by calculus
B-beyond connective tissues in the junctional epithelium
C-at the end of the junctional epithelium
D-Touching the middle of junctional epithelium
E-Touching sulculuar epithelium
------------------------------------------------------------------------------------
279 A 58 year old male has been treated with radiation for carcinoma of tongue. The
patient complains of pain associated with
poor dentition. The dental management would be:
A-Immediate extraction of any poor teeth under local anaesthetic with antibiotic coverage
B-Segmental dental clearance and closure to eliminate problems
C-No dental treatment may be due to neuronic of neoplasms
D. Clearance of poor dentition followed by hyperbaric oxygen treatment plus a primary
closure of wounds under antibiotic
coverage
E. No extraction as radionecrosis is an important sequelae
-------------------------------------------------------------------------------------------
280 Which of the following is NOT true about anticoagulation therapy?
INR of 3 is enough to start any extraction
A-Affects extrinsic system and increases prothrombin time
B-Heparin can be given subcutaneously and acts rapidly
C-It takes at least 12 hours for Vitamin K to reverse the effects of coumarin
-------------------------------------------------------------------------------------------
2811 n a radiograph the roots of the upper teeth are too short because of:
Inadequate horizontal angulation
A-Too high vertical angulation
B-Too small vertical angulation
C-Parallel
-----------------------------------------------------------------------------------------
282 Characteristic of Squamous Cell Carcinoma of the tongue
A-more in white skinned people
B-more in alcohol drinking smoking males
C-associated with Plummer-Wilson-Syndrome
--------------------------------------------------------------------------------------------
283 Characteristic of Squamous Cell Carcinoma of the lips
A-It reacts far simply to radiotherapy
B-metastizes mainly by blood (lymph and local is right for metastasis)
C-relatively rare in Australia
D-Relatively high in Australia
-------------------------------------------------------------------------------------------
284 Which type of dentin is not formed due to pulp pathology?
A.Reparative dentin
B.Secondary dentin
C.Primary dentin
D.Reaction dentin
E.Tertiary dentin
---------------------------------------------------------------------------------------------
285 Which is not true in sickle cell anaemia?
A.Deformed cells with less oxygen transport capacity
B.Higher infarction risk
C.Have wide bone marrow spaces with narrow trabeculae in the alveolar bone of oral
cavity
D.Resistant to malaria parasites
E.More common in mediterranean people
--------------------------------------------------------------------------------------------
286 Normal prothrombin time and elevated partial thromboblastin time is seen in
A.Factor VIII deficiancy
B.Thrombocytic pupura
C.Leukemia
D.Von Willebrand disease

---------------------------------------------------------------------------------------------------

287 Which of the following describes best a 9 years-old child permanent dentition?
A. 1612 11 I 212226
464241 I 313236
B. 12 11 I 2122
4241 I 3132
--------------------------------------------------------------------------------------------
288 What is the best reason to promote tooth brushing to the public?
A.Less fissure caries
B.Less gingivitis
C.Gingival massage
---------------------------------------------------------------------------------------------
289 What is untrue about diabetes?
A.Hypoglycaemia is more common than hyperglycaemia
B.lnsulin-dependend patients are of more concern than non insulin-dependend
c. Adrenalin causes a decrease in the blood glucose level (adrenaline raises sugar)
----------------------------------------------------------------------------------------

290 Which of the following is a feature of Streptococcus mutans?


A.lt does not require a special environment to grow
B.lt can be easily transported from one part of the oral cavity to another
C.lt has the ability to re structure carbohydrates
-----------------------------------------------------------------------------------------
291 What does the term "caries prevalence" mean?
A.The total number of carious areas affected and any present caries
B.The individual risk for a patient to acquire caries
----------------------------------------------------------------------------------------
292 Which of the following are features of herpetic gingivostomatitis?
A-Irritability
B-Fever
C-Occurs in teenagers
D-Vesicles occur only on buccal mucosa and tongue
--------------------------------------------------------------------------------------------
293 A patient in your dental chair suddenly becomes agitated with shallow breathing, full
pulse and a blood pressure of 150/80.
You would
A.Give oxygen
B.Give insulin
C.Give glucose
D.Place patient in supine position
--------------------------------------------------------------------------------------------
294 The principle clinical sign of active bruxism is:
A-Head and / or neck pain
B-Excessive tooth wear
C-Temporomandibular joint clicking
D-Sensitive teeth
----------------------------------------------------------------------------------------------
295 An adult patient attends your practice complaining of pain-and swelling aSsOCiated
with a previously restored upper first premolar tooth. The pain has been present for a
number of days and is no longer responding to analgesics. His dentition is otherwise well
maintained and his periodontal health is good.
What is the most appropriate approach to treatment?
A-Antibiotics and analgesics.
B-Extract the tooth
C-Carry out a pulpotomy and temporary dressing.
D- Drainage
--------------------------------------------------------------------------------------------
296 A patient reports that his post crown has fallen out. This crown had been present for
many years. You note that there appears to be a hairline vertical fracture of the root. The
tooth is symptomless.
What is the most sensible approach to treatment?
A-Replace the post crown using a resin-reinforced glass ionomer material
B-Replace the post crown using a polycarboxylate cement
C-Replace the post crown using a dentine bonding agent and a resin-reinforced glass
ionomer material
D- Replace the post crown using a resin composite luting agent
E. Arrange to extract the tooth
---------------------------------------------------------------------------------------------
297 A patient says that he does not like the appearance of his previously root filled upper
central incisor tooth. His dentition is otherwise well maintained and his periodontal health
is good. The tooth appears to be darker than the adjacent teeth. What is the most
appropriate approach to treatment?
A-Provision of a post crown
B-Provision of an all ceramic crown
C-Provision of a metal bonded to ceramic crown
D- Non vital bleaching
E- Provision of a porcelain veneer
---------------------------------------------------------------------------------------------
298 A 25 year old male attends for the first time complaining of sensitivity of a number of
teeth. On examination, the occlusal surfaces of all the teeth are worn with obvious wear
facets on the canines and premolars. Posterior amalgam restorations are proud of the
surrounding tooth.
What would be the first stage management?
A-Take impressions for study models
B-Prescribe fluoride mouth rinse
C-Replace the amalgam restorations
D-Dietary analysis
----------------------------------------------------------------------------------------------

299 A 35 year old male patient who admits to grinding his teeth at night has a number of
wedge-shaped cervical (Class V) lesions on his upper premolar teeth. These are causing
sensitivity and are approximately 3mm deep. What is the correct management option?
A-Provide tooth brushing instruction and fluoride
B-Restore the lesions with compomer
C-Restore the lesions with micro-filled composite
D-Restore the lesions with a hybrid composite
E-Restore the lesions with conventional glass-ionomer
-------------------------------------------------------------------------------------------

300 A patient attends with pain of four days duration in a carious upper molar tooth. The
pain is constant and is not relieved by paracetemol. Sleep has been disturbed by the pain.
The tooth is tender to percussion and gives a positive response to Ethyl Chloride. What is
the most likely diagnosis?
A-Pericoronitis
B-Apical periodontitis
C-Marginal periodontitis
D-Reversible pulpitis
-------------------------------------------------------------------------------------------
301 A 14 year old patient attends with a decayed and hypoplastic LL7. A radiograph shows
the presence of an unerupted LL8 and the LL6 is sound. What would be the most
appropriate long-term treatment for this tooth?
A-Amalgam restoration
B-Antibiotics
D- Root canal therapy (best is to extract)
E-Sedative dressing
--------------------------------------------------------------------------------------------
302 A 30 yr-old patient attends complaining of pain from the lower left quadrant. Clinical
examination reveals a dentition with generally good oral hygiene. There is no significant
periodontal pocketing other:.than an isolated defect in the region of the furcation of lower
left first molar which is non-mobile. The gingival tissue in thiS area appears erythematous
and slightly hyperplastic with a purulent exudate on probing. From the list below, which is
the most appropriate next step?
A-Obtain a radiograph
B-Biopsy the gingival tissue
C-Remove the restoration
D-Vitality testing
E-Prescribe antibiotics
--------------------------------------------------------------------------------------------
303 A 40 yr old patient had root-canal treatment to his upper first molar. This was
performed 6 months ago using contemporary techniques under rubber dam and was
crowned after completion of treatment. He attends complaining of continued discomfort
from this tooth. Radiographic examination shows each of the three roots to be obturated
with a well-condensed filling to the full working length though there is no evidence of in-fill
of the periapical lesion when compared to the pre-op view. What is the most
likely cause of the continued problem?
A-Extra-radicular infection
B-Contamination of canal(s) with E.faecalis
C-Uninstrumented canal
D-Ve rtica I root fra ctu re
E-Perio-endo problem
------------------------------------------------------------------------------------------
304 A 21 year old female presents for the first time to your practice. She is very upset with
the appearance of her upper left central incisor. On examination you find healthy oral hard
and soft tissues and excellent oral hygiene. On close examination you can see that the upper
left central incisor is slightly greyer than the upper right central incisor and has a
composite restoration placed palatally. What is the most appropriate form of treatment
given the information you have?
A-Bleaching with carbamide peroxide in custom formed trays of upper and lower arches
B-A bonded crown
C-A composite veneer
D-A porcelain veneer
E-Non vital bleaching with carbamide peroxide
----------------------------------------------------------------------------------------
305 EDTA (ethylene diamine tetra-acetate) has useful roles in certain situations in clinical
dentistry. When would you use EDTA?
A-As a pulp capping agent
B-As a root canal irrigant( correct answer)
C-As a mouthwash
D-As a root end filling material
E-As a dentine bonding agent
-----------------------------------------------------------------------------------------

306 A patient presents with a history of clicking from their temporomandibular joint. This
click occurs mid way through the opening cycle and is consistent. There is some pre-
auricular pain and the lateral pterygoid muscle on the affected side is tender to resisted
movement test. There is no trismus and the click is not present when the patient opens
from an incisor edge to edge relationship, instead of her normal Class I occlusion. The
patient would like treatment.
The most appropriate occlusal splint for this patient would be:
A-Stabilisation splint
B-Localised Occlusal Interference Splint
C-Bite Raiser
D-Soft Bite Guard
E-Anterior Repositioner Splint
-----------------------------------------------------------------------------------------
307 A patient presents with a history of a post-crown having fallen out. The post-crown
was originally placed fifteen years ago and had been successful up until four months ago
since when it has come out and been recemented four times. At recementation there was
no evidence of any caries. The patient had been a regular attender and not needed any
restorative treatment for the last eight years.
Which of the following is the most likely cause for the failure of this crown?
A-The post was to narrow
B-The post was too short.
C-The root canal treatment was failing.
D-A vertical root fracture was present.
E-There were excessive occlusal loads on the tooth.
------------------------------------------------------------------------------------------
308 A 23 year old patient attends complaining of pain in an upper right molar and is keen
to keep the tooth. The pain is typically sharp in nature, is triggered by cold and persists
after removal of the cold stimulus. The tooth is not tender to percussion; a radiograph of
the upper right first molar shows a large radiolucency extending to the pulp horn but no
peri-radicular changes.
What treatment is most likely needed in this case?
A-Oral pygiene instruction and fluoride application
B-Excavation of caries and placement of a permanent restoration
C-Root -canaI treatment
D-Indirect pulp cap and restoration
E-Direct pulp cap and restoration
---------------------------------------------------------------------------------------------
309 A patient attends your surgery complaining of severe pain, swelling and mobility
associated with a lower first molar tooth in which there is a broken filling. A periapical
radiograph indicates that the tooth has not been root filled and there is loss of apical
lamina dura associated with the distal root and at the bifurcation. The periodontal bone
support is good. There is no significant pocketing.
What is the likely diagnosis?
A-Chronic periapical periodontitis
B-An acute periodontal-endodontic periodontitis
C-A chronic periodontal-endodontic periodontitis
D-An acute periapical periodontitis
E-Pararadicular periodontitis
---------------------------------------------------------------------------------------------
310 An upper incisor in a 16 year old patient has suffered trauma and the coronal tissue
has been lost. The tooth has been endodontically treated. How is the tooth best restored?
A-With a fibre post, direct core and crown.
B-With a direct core and crown.
C-With composite
D-With an indirect post-core from a pre-fabricated pattern and a crown
E-With a custom indirect post-core and crown
----------------------------------------------------------------------------------------------
311 A 50 year old male patient has a Class III jaw relationship with an anterior open bite. It
is planned to restore his lower right second molar, which has suffered tooth wear and
fracture, with an indirect restoration. This tooth has approximately 2mm of coronal height.
What would be the most suitable approach to restore this tooth?
A-Provide an adhesively retained gold onlay
B-Provide a conventional full crown
C-Increase the vertical dimension and provide a full crown
D-Surgically crown lengthen and provide a gold crown
E-Provide an adhesively retained ceramic onlay
-------------------------------------------------------------------------------------------
312 A patient complains of a lower incisor which has been mobile for several months. The
radiograph indicates a normal level of bony support although the periodontal space has
widened. The apical bone appears normal. The tooth is tender to pressure. Which of the
following tests and or examinations would be most likely to provide a diagnosis?
A-Masticatory muscle palpation
B-Electric pulp test
C-Occlusal examination
D-Ethyl chloride test
E-Hot gutta percha application
-------------------------------------------------------------------------------------------
313 A 23 year old male presents to your surgery. He lost his upper lateral incisors some 10
years ago in a swimming pool accident. Since then he has been wearing a 'spoon' denture
which he now feels in aesthetically unacceptable. He has sought an opinion on dental
implants but has been told that he would need bone grafting for this to be successful and he
is not prepared to undergo this. His dentition is excellent with no restorations and a Class I
occlusion. He wants some advice on what the best treatment might be. Which option would
you put first on your list of possibilities?
A-Two fixed - fixed resin bonded bridges using the central and canine teeth
B-Two cantilever resin bonded bridges from the central incisors and canines.
C-Two conventional fixed - fixed bridges from the canine
D-Conventional cantilever bridges from the canines
E-Cobalt chrome partial denture
---------------------------------------------------------------------------------------------
314 A stabilisation splint [Michegan splint] is commonly indicated in patients needing
advanced restorative dentistry and patients suffering from some Temporomandibular
Disorders Other than upper and lower impressions, which of the following records will be
needed to construct this splint?
A-Facebow and a protrusive wax record
B-Facebow and an "Oclusal Sketch"
C-Centric Relation [Retruded Contact Position] record
D-Facebow and a Centric Occlusion [Incuspation Position] record
E-Facebow and Centric Relation [Retruded Contact Position] record
---------------------------------------------------------------------------------------------
315 A 55 year old female patient is missing her upper right second premolar and upper
right first molar and also is missing the upper left second molar. The upper right second
molar is functional and has an amalgam restoration (MOD and buccal wall) that requires
replacing. The patient has no functional or aesthetic concerns.
What would be the treatment of choice in this situation?
A-Provide an upper removable partial denture
B-Replace the amalgam in the upper right 7 only
C-Provide a full coverage crown in the upper right 7
D-Provide a fixed bridge in the upper right quadrant
E-Provide a full coverage crown in the upper right 7 with guide planes and occlusal rests
--------------------------------------------------------------------------------------------

316 Endodontic treatment has failed on an upper first molar; the patient is keen to retain
the tooth. There are persistent symptoms from the tooth. Radiographically there is
evidence of periapical radiolucency although the three canals are obturated with good
length and compaction.
What is the best course of action?
A-Extract the tooth
B-Re-treatment with an iodine solution as irrigant
C-Re-treatment with hypochlorite solution as irrigant
D-Re-treat the tooth, looking particularly for additional canals
E-Surgical apicectomy
------------------------------------------------------------------------------------------
317 You suspect that there is occlusal caries in the lower right first permanent molar of a
10 year old child. You wish to confirm your suspicions. Which diagnostic test is most
commonly used in this situation?
A-Bitewing radiography
B-Electro-conductive caries monitors
C-Fibro-optic transillumination
D-Panoramic radiography
E-Visual examination of a dried tooth.
------------------------------------------------------------------------------------------
318 You notice that a 20 year old patient has marked tooth surface loss associated with the
labial and palatal aspects and incisal edges of the upper anterior teeth. They are sensitive
to hot and cold. The remainder of the dentition is mainlyunaffected. What is the likely
diagnosis?
A-Attrition
B-Active erosion
C-Passive erosion
D-Abfractions

---------------------------------------------------------------------------------------------------------------------
---------------------------------------------------------------------------------------------------------------------
---------------------------------------------

321 In the preparation of Premolar class I cavity what is the best way of getting
retention,
A-Slightly done undercut of the mesial and distal walls
B-Slightly done undercut of the buccal and lingual walls
C-The convergence of the cavity walls
-------------------------------------------------------------------
323. After the placement of a class I amalgam the patient comes back to you
complaining of pain on masticating and biting; what is the first thing you would look at,
A-Occlusal height
B-Contacts areas
-----------------------------------------------------------------------------------
324. Child comes to your clinic with a fractured incisor 3 mm super-gingival, how would
you treat the case,
A-Formocretasol pulpotomy
B-Calcium hydroxide pulpotomy
C-Pulpectomy
D-Direct capping
E-Indirect capping
--------------------------------------------------------------------------------------
325. What is Ante's Law about,
A-The relation between the span of the bridge and the poetics
B-The periodontal area of the abutment teeth
C-The relation between the length of the root and the abutment.
--------------------------------------------------------------------------------------
326. What is the best way to cement Maryland bridge,
A.GIC
Resin
High compression restorative resin
Zinc Phosphate cement
Oxide Zinc and eugenol

------------------------------------------------------------------------------------
327. The ideal length of core in the fabrication of crown and core of endodontically
treated tooth is,
A-1.5 of crown length
B-The length of the crown
C-2/3 tooth/root length
D. 1/2, root length
-------------------------------------------------------------------------------------

328. If aesthetic is not a concern what is the first thing to do to treat soreness under
dentures,
A-Take the denture off for a week
B-Rinse the denture in nystatin
C-Apply tissue conditioner
------------------------------------------------------------------------------------------

329. While you finishing a class I cavity, the enamel is sound but you noticed in the
dentine and on the Dento-enamel junction a
brown line, what is your response,
A. You leave it and complete the final restoration
B.You extend you preparation and clean it
C.You apply a cover of varnish
--------------------------------------------------------------------------------------------
331. What is the property of high copper amalgam
A-Reduced physical creep
B-Higher retention
C-increase strength
D-Decrease strength
E- Decrease setting time
-------------------------------------------------------------------------------------------
332. In regards to colours what is Chroma stands for,
A-Degree of saturation of hue
B-Brightness
C-Value
D-Contrast
---------------------------------------------------------------------------------------------
333. Frankfort plane extends from,
A-horizontally from Sella to nasion
B-Horizontally from point on superior aspect of external auditory meatus to orbitale
-------------------------------------------------------------------------------------------
334. Which of the following local anaesthetic is indicated in case of the need to long
acting one after a surgical operation,
A. Lidocaine
B.Mepivacaine
C.Bupivacaine
----------------------------------------------------------------------------------------
336. Which one of this restorative method will be LEAST compromised by a core,
A-Amalgam
B-Composite
C-GIC
D-Cast gold
--------------------------------------------------------------------------------------------
337. In preparing a very small proximal amalgam cavity on a molar tooth what would
consider,
A-Extend the cavity to the gingival margin
B-Extend the cavity beyond the contacts areas
C-Achieve at least 2mm in dentine
D-Extend cavity just beyond dento enamel junction
------------------------------------------------------------------------------------------
338. What is true about partial dentures,
A-They cause an immediate changes in the oral plaque behaviour
B-Night wearing of dentures reduces plaque accumulation
C-Relieving the gingival area reduces the gingival enlargement.
--------------------------------------------------------------------------------------------
339. The biting load of denture base to tissues compared to teeth are,
A-Ten times more
B-Ten times less
C-Twenty times less
D-Twenty times more
E-Equal
-------------------------------------------------------------------------------------------

340. Compound is,


A-Very accurate compression material
B-Thermoplastic material
-----------------------------------------------------------------------------------------
342. A patient comes to you with medium pain of tooth filled with Composite resin as a
result of cold or hot drinks, what your initial
management will be,
A-Remove the restorative material and start an Endontic treatment
B-Remove the restorative material and place a sedative temporary material
C-Place a coat of bonding material on the old composite

-----------------------------------------------------------------------------------------

343. Throbbing pain increases with heat and cold stimuli, the MOST probable diagnosis
is,
A-Cyst
B-Occlusal trauma
C-Advanced pulpitis
----------------------------------------------------------------------------------------
344. In making your custom trays which of the following is true,
A -uniform thickness is required
B-Perforation is better
C-Only adhesive is better than perforation
--------------------------------------------------------------------------------------
345. The most common cause of caries in children is,
A-Soft diet
B-High intake of carbohydrate
C-Poor oral hygiene

----------------------------------------------------------------------------------------
346. The best storage media for avulsed tooth is,
A-Saline
B-Milk
C-Water
D-Saliva
--------------------------------------------------------------------------------------------

347. An occlusal approaching clasp TIP,


A-Should occupy a predetermined undercut
B-Contact the tooth under the survey line
C-Rigid
------------------------------------------------------------------------------------------

348. In the construction of partial denture the surveyor is not used to,
Contour the wax as part of the fabrication of the working cast
A-Locate the guide planes
B-Determine the location of indirect retainers
C-Identify any undesirable undercuts

------------------------------------------------------------------------------------------

349. The main purpose of finishing the enamel walls is,


A-Remove loose enamel rods
B-Provide a better surface for the adoption of restorative material
--------------------------------------------------------------------------------------------

350. A female patient comes to you complaining of persistent pain in heavily restored
central incisor; you suspect pulpitis and you
have been told that she is in transit leaving by plane next day. Your treatment will be,
A-Remove filling and place a sedative dressings
B-Pulp extirpation and obturate with Ledermix dressings
C-Prescribe analgesics and systemic antibiotic
--------------------------------------------------------------------------------------------

351.which of the following is true regarding TMJ dysfunction,


A-It is always due to arthritis, should be treated with NSAIDS before attempting surgery
B-Raising bite increases the space in the joint and should be attempted before surgery
C-It is mostly due to the medial movement of the condylar head over the glenoid fossa

----------------------------------------------------------------------------------------------
352.The location of Class V is in,
A-The buccaI pit /fissu re
B-The occlusal surface
C-The cervical third
----------------------------------------------------------------------------------------------
353. Occlusal cavity with extension of the buccal fissure is classified as,
A-Class II
B-Class III
C-Class I
-------------------------------------------------------------------------------------------
354. Which of the following does not affect the elasticity of retentive clasp?
Length of the arm
A-The cross section shape
B-The material used
C-The undercut area

------------------------------------------------------------------------------------------
355. Following calcium hydroxide pulpotomy, the dentist would expect dentine bridge to
form at,
A-The exact level of amputation
B-Level some where below the amputation
C-Half way between amputation and apex
D-At the apical region of the tooth

--------------------------------------------------------------------------------------------
356. In the construction of a full veneer gold crown, future recession of gingival tissue
can be prevented or at least minimised by,
A-Extension of the crown 1 mm under the gingival crevice
B-Reproduction of normal tooth incline in the gingival one third of the crown
C-Slight over contouring of the tooth in the gingival one fifth of the crown
D-Slight under contouring of the tooth in the gingival one fifth of the crown

------------------------------------------------------------------------------------------
357. A partial denture that seats on the master cast but fails to seat correctly in the
mouth is a result of,
A-Contraction of the metal framework during casting
B-Insufficient expansion of the investment material
C-Distortion of impression
D-Failure to block out unwanted undercuts
--------------------------------------------------------------------------------------------

358. Which of the following muscles may affect the borders of mandibular complete
denture?
A-Mentalis
B-Lateral pterygoid
C-Orbicularis oris
D-Levator oris
E-Temporalis

---------------------------------------------------------------------------------------------
360. What is the main purpose of using corticosteroids in pulpal obturation material?
A-For their antibiotic action
B-For their anti inflammatory action
C-To relief pulp pressure
---------------------------------------------------------------------------------------
361. Which of the following statements is incorrect regarding Smoker's Keratosis?
A-Typically affects the hard palate
B-Minor mucous glands are swollen with red orifices
C-There is a little regression if smoking is stopped
---------------------------------------------------------------------------------------
362. 13 years old boy comes to you with excessive hyperplasia of the gingiva as a
result of Phenytoin what is your management,
A-Stop the medication
B-Force a strict oral hygiene and surgical removal of excess gingival tissues
C-Debridement and conservative approach
--------------------------------------------------------------------------------------------
363. White man 56 years old comes to you with a brown spot on his gingiva and
another one on his oral mucosa, when taking the history he mentioned a weight and
memory lost. He as well complains of headaches. What is your most probable diagnosis,
A-Addison's disease
B-Hyperthyroidism
C-Hypothyroidism
--------------------------------------------------------------------------------------------
364. While removing the second primary molar of 9 years old child, the apical Y. of the
root fracture and stay in the socket,
A-You will just leave it and observe it
B-You take surgically by a lingual flap
C-You try to take out by using a root apex elevator
D-You use a fine end forceps to take it out
----------------------------------------------------------------------------------------------
365 What is the most important factor to reduces dental irradiation,
A-Speed of film
B-Collimation
C-Filtration
D. Cone shape and length
----------------------------------------------------------------------------------------------
366. With view to Nitrous Oxide what is the major pharmacological problem?
A-Contraindicated in pregnancy
B-Contra indicated in cardiac dysrhythmias
----------------------------------------------------------------------------------------------
367. Which of the following is an expansile lesion of the oral mucosa,
A-Keratocyte
B-Radicular cyst
C-Cementoma
--------------------------------------------------------------------------------------------
368. The concentration of Fluoride in the topical NaF
A.2%
B.5%
C. 8%
D.10%
--------------------------------------------------------------------------------------------
369. patient in your dental chair shows chest pain, weak pulse and dysponea, what is
your initial management,
A-Give a nitro-glycerine tablet and keep the patient up seated'
B-Put the patient in supine position
C-Do nothing and wait until the symptoms go
-------------------------------------------------------------------------------------------
370. What are two teeth connected at the cementum called,
A. Concrescence
B. Dilaceration
C. Gemination
D. Fusion
------------------------------------------------------------------------------------------
371. Developer contaminated with other chemical and was not mixed pro perlv. What is
the effect on the X-ray film?
A-Too dark film
B-Light film
C-Foggy
---------------------------------------------------------------------------------------------
372. Which part of the cranium is considered as the most stable area,
A-Frankfort plane
B-Occlusal plane
C-Anterior cranial plane
D-Anterior nasal to gnathion.
--------------------------------------------------------------------------------------------
374. Branchial Cleft cyst is located,
A-Medial to the neck
B-On anterior border of the Sternocleidomastoid muscle
C-Shows when swallowing
-------------------------------------------------------------------------------------------
375. the most common way of oral carcinomas to other tissues is/are
A-Lymphatic
B-Invasion and blood
C-Lymphatic and invasion
D-Blood and lymphatic
----------------------------------------------------------------------------------------
376. The main purpose of periodontal treatment is,
A-Elimination of plaque and calculus
B-Elimination of periodontal pockets
C-Reformation of all the periodontal ligaments
D-The elimination of all occlusal trauma
----------------------------------------------------------------------------------------
377. What is the approximate un stimulated salivary flow rate,
A-2 ml/min
B-0.2 ml/min
C- 0.02 ml/min
D. 20 ml/min
------------------------------------------------------------------------------------------
378. The thermal and electric pulp tests will,
A-Give an accurate indications of the pulp status
B-The patient's response will be either pain or no pain
C-The patient can differentiate between cold or hot stimuli
-------------------------------------------------------------------------------------------
379. Immediately after the extraction of lower molar the patient complains of post
operation bleeding and pain, how would
manage this,
A-Prescribe analgesics and ask the patient to follow a strict oral hygiene
B-Administer 5% Marcaine Local Anastatic, prescribe analgesics and pack the socket
with alvogyl
C-Administer 5% Marcaine Local Anastatic, suture the socket and prescribe analgesics
D-Suture and give pressure packs
---------------------------------------------------------------------------------------------
380. Gracey curette is characterized by,
A-The blade and the shank form 90 angle
B-Can be used on both sides
C-Can be used on any tooth surface
D-It is specific for each surface of the tooth
--------------------------------------------------------------------------------------------
381. The removable partial denture requires relining what is would be the most
appropriate action,
A-take a new impression by asking the patient to occlude on it
B-Provide equal space between denture and gingival tissues.
C-Make sure the framework and retainers are seated in place before taking impression
-----------------------------------------------------------------------------------------
382. In regards to dentine strength, which is the right sequence,
A-Affected dentine> Sound dentine> Infected dentine
B-Sound dentine> Affected dentine> Infected dentine
--------------------------------------------------------------------------------------
383. Symptoms free patient comes to you after four weeks of an endodontic treatment
and you find on radiograph the canal is over filled with what it seems to be a cone of
Gutta Percha 1mm beyond the apex with a radiolucent small area. What is your initial
management?,
A-Start apiectomy through a flap and surgery
B-Obturate the root canal
C-Ask for a recall and observe in three months time
D-Seal the pulp chamber and keep it under observation
--------------------------------------------------------------------------------------------
384. After obturation and on X-ray you notice the obturation materials are lmm beyond
apex. What is your first management?
A-Refill the canal
B-Pull the GP cone about lmm out and take a new X-ray
C-Leave it as it
-------------------------------------------------------------------------------------------
385. 2.21mg NaF contains,
A. lmg fluoride
B.2 mg
C. 0.5 mg
D- 3 mg
------------------------------------------------------------------------------------------
387. Four years kid shows at your clinic with open bite as a result of thumb sucking, you
notice a delayed speech ability what would
be your first management,
A-Refer to a speech therapist
B-Apply a removable habit inhibitor denture
C-Apply a removable habit inhibitor denture and educate the parents about it so the kid
will not be taking it off so often
---------------------------------------------------------------------------------------
388. Two central incisors on a radiograph are showing with what looks like eye drop
radiolucency. You decided to start endodontic treatment on these teeth but you tried to
open access to the root canal you find clearly closed orifices with what look like
secondary dentine. What is your initial management?
A-Leave as it and start a permanent restoration.
B-Start systemic antibiotic
C-Try to ream and file canals

390 when probing for periodontal disease the tip of the probe will be,
A-At the coronal end of junctional epithelium
B-At the top of the gingival calculus
-------------------------------------------------------------------------------------------
392. The most desirable outcome of endodontic treatment is,
A-The healing of the alveolar bone
B-The deposition of cementum at the apex
C-Formation of fibrous capsule around the apex
--------------------------------------------------------------------------------------
393. What is NOT related to the normal aging process?
A-Progressive bone loss
B-Reduced elasticity of muscles
C-Decreased elasticity of the skin
D-Lower pain threshold
----------------------------------------------------------------------------------------
394. The most accurate finding of pulpal pathology
A-Radiolucency on the apical region
B-Pain on hot or cold drinks
C-The absence of response to pulp testing
--------------------------------------------------------------------------------------
395 For dental caries to progress in dentine,the dentine must contain soluble collagen
A-enamel must contain glycoproteins
B-diet must contain simple carbohydrate
C-diet must contain polysaccharides
D-pulp must contain complement
------------------------------------------------------------------------------------
396 A patient on the dental chair has cardiac arrest which is INCORRECT,
A-Observing the vital signs and check that the air way is clear is at high importance
B-Expired air has 15% 02 only, and cardiac compressions achieve 30-40% of cardiac
output
C-Intermittent positive pressure at the rate of 40/min will reduce the chances of cerebral
hypoxia
D-Intermittent positive pressure is better than mouth to mouth when it has been given at
the same rate.
----------------------------------------------------------------------------------------
397. A 9 year old boy has a small white discolouration on his maxillary central incisor. The
lesion is most probably,
A-Hypocalcification due to trauma of the primary predecessor
B-Hypoplasia due to acute systemic infection when 6-12 months old
C-Defect during the histo differentiation stage of development
D-Defect during the morpho differentiation stage of development
---------------------------------------------------------------------------------------------
398. The best method to take X-ray of the maxillary sinus is,
A-Periapical radiograph
B-Panoramic view
C-Lateral cephaloghraph
D-Occipitomental view
E. Reverse Towne's view
------------------------------------------------------------------------------------------
399. Which of the following is not a part of the fully formed enamel organ,
A-Outer enamel epithelium
B-Inner enamel epithelium
-----------------------------------------------------------------------------------------
400.18 years old female her weight is 52Kg and she is 163cm tall. On dental examination
erosion of teeth on the most of her
lingual surfaces is clearly showing. Dietary history revealed a daily rate of 5000 to 7000
Kcal/day. What is most probable would be
her case?
A-Alcoholism
B-Drug abuse
C-Bulimia
D-Excessive smoking
E-Diabetic mellitus type I
-------------------------------------------------------------------------------------------
401. Which one of the following is true in regards t osseointegration implants in dentistry?
A-Fibrous tissues are formed and integrated directly between titanium and bone
B-Following insertion, implants can be immediately loaded without problem
C-The success of the implants is directly proportional to its area of contact with bone
D-The success of the implants depends mostly on low torque preparation and insertion of
the fixture
-----------------------------------------------------------------------------------------
403. On X-ray, the buccal roots of 16 is considerably elongated; this is a result of,
A-Too great vertical angulation
B-Inadequate vertical angulation
C-Excessive object film distance
---------------------------------------------------------------------------------------
404. The principle muscle responsible for the opening of the mouth is,
A-Mylohyoid
B-Anterior temporal
C-Posterior temporal
D- Anterior belly of digastrics
---------------------------------------------------------------------------------------
405. Denture stomatitis is commonly associated with,
A-The continuous wearing of removable orthodontic appliances in otherwise healthy
patient
B-The proliferation of hypertrophic tissue at the denture periphery
C-The overgrowth of some constituents of oral normal microflora
D-Allergy to denture base material
---------------------------------------------------------------------------------------
406. Which of the following is NOT characteristic of Down's syndrome?
A-Decreased neutrophil function
B-Macroglossia
C-Macrodontia
D-An increased susceptibility to periodontal disease
E-Congenitally missing teeth
---------------------------------------------------------------------------------------

407. Which of the following is the best evidence that a previous periodontal treatment is
successful?
A-The patient keeps a 3 month recall appointment
B-There is no extrinsic stain
C-The patient demonstrates good understanding of brushing and flossing techniques
D-There is no bleeding on probing
-------------------------------------------------------------------------------------
408 A 65 year old female presents at your surgery complaining of an extremely sharp pain
of a few seconds duration which arises whenever she touches an area of skin above the
right hand side of her upper lip adjacent to the angle of her mouth. The patient is fit and
well and is not taking any medication. You make a diagnosis of trigeminal neuralgia.
What is the drug of choice for treating trigeminal neuralgia in such a patient?
A-Carbamazepine
B-Oxcarbamazepine
C-Gabapentin
D-Phenytoin
E-Ibuprofen

----------------------------------------------------------------------------------------
409 A patient on examination was found to have swollen gingiva around a crown that had
been present for several years. The
papillae were particularly enlarged.
What is the most important feature of a crown that may be responsible for this?
A-Material of the Crown
B-The occlusion
C-proximal Contour
D-Labial Contour
E-Surface finish
-------------------------------------------------------------------------------------------
410 A 13 year old girl presents with an unerupted permanent canine and a retained
primary canine. You cannot palpate the
unerupted canine in the buccal sulcus and you are uncertain as to whether it is displaced
palatally or in the line of the arch.
Which single radiographic view would be most helpful in locating the unerupted tooth?
A. Bitewing
B.DPT
C.Lateral oblique
D.Single periapical
E.Upper anterior occlusal
-------------------------------------------------------------------------------------------
411 A 30 year old man with unknown allergy to latex goes into anaphylactic shock whilst
being treated in the dental surgery.
Which drug and route of administration is of most benefit in this situation?
A-Hydrocortisone - orally
B-Chlorphenamine - intramuscularly
C-Chlorphenamine - orally
D-Epinephrine - intravenously
E-Epinephrine - intramuscularly
----------------------------------------------------------------------------------------
412 A 62 year old female presents at your surgery complaining of a persistent, dull ache
affecting her upper left 4. The pain is
present all the time but varies in its severity although the patient cannot think of any
exacerbating factors. It is not relieved by
analgesics. Over the past 18 months the patient has had several teeth extracted from the
upper left quadrant. Each extraction
brings about temporary relief of her symptoms only for them to recur in an adjacent tooth.
What is the most likely cause of the patient's pain?
A-Acute/reversible pulpitis
B-Dentine sensitivity
C-Chronic/irreversible pulpitis
D-Atypical odontalgia
E- Trigeminal neuralgia
------------------------------------------------------------------------------------
413 A 25 year old patient attends your surgery complaining of a swelling at the angle of the
mandible. A radiograph shows a uni-
locular radiolucency associated with the crown of an unerupted wisdom tooth.
Which of the following is the most likely diagnosis?
A-Radicular cyst
B-Dentigerous cyst
C-Lateral periodontal cyst
D-Ameloblastoma
E-Odontogenic keratocyst
-------------------------------------------------------------------------------------
414 A 70 year old female, who suffers with persistent looseness of her lower complete
denture, is considered for implants. She will
require radiological evaluation of the potential implant sites.
Which would be the most appropriate radiological investigation at this stage?
A-MRI scan of the mandible
B-CT scan of the mandible
C-Periapical radiographs of the mandibular anterior region
D-True lower anterior occlusal view
E-Panoramic radiograph
----------------------------------------------------------------------------------------
415 Six months ago you saw a child patient, then aged 9 years. His upper right maxillary
canine was palpable in the labial sulcus but the upper left was not. The situation is now
unchanged, so you have taken two periapicals of the non-palpable tooth. They both show
that there is some resorption of the CI root but the permanent canine appears somewhat
mesially angled and is more mesial on the more mesially positioned film.
What is your the best course of action?
A-Keep a careful watch on it and take another x-ray in 6 months.
B-Refer to an oral surgeon for early exposure of the permanent canine.
C-Refer to an oral surgeon for early removal of the permanent canine before it damages the
lateral incisor.
D-Refer to an orthodontist for a treatment plan
E-Wait and watch' until the child is 11.
-----------------------------------------------------------------------------------------
416 An apical radiolucency (2mm) is noticed as an incidental radiographic finding
associated with the apex of the mesiobuccal root of the lower right first molar. The tooth
has been root filled but is 2mm short of the radiographic apex. There are no other clinical
or radiographic findings and the patient is fit and well. What is the most appropriate course
of action?
A.Extract the tooth.
B.Redo the root filling
C.Perform periapical surgery.
D.Advise the patient of the situation and monitor clinically and radiographically
E.Prescribe antibiotics then review
-------------------------------------------------------------------------------------------

416 An apical radiolucency (2mm) is noticed as an incidental radiographic finding associated


with the apex of the mesiobuccal root of the lower right first molar. The tooth has been root filled
but is 2mm short of the radiographic apex. There are no other clinical or radiographic findings
and the patient is fit and well. What is the most appropriate course of action?
A.Extract the tooth.
b.Redo the root filling
c.Perform periapical surgery.
d.Advise the patient of the situation and monitor clinically and radiographically
e.Prescribe antibiotics then review
---------------------------------------------------------------------------------------------
417 A 43year old patient is missing on the upper right the first premolar and molar. He has good
oral hygiene and requests a fixed replacement for these teeth. The other teeth on the same side
are all moderately restored with MOD amalgam restorations and are vital, except the canine,
which has a very large restoration and is root-filled. He has group function. Radiographs show a
large sinus cavity and no peri-apical pathology. What would be the restoration of choice for
replacement of the missing teeth?
A-Implant supported crowns
B-A conventional fixed bridge using the 7 and 5 as abutments
C-Two conventional cantilevered bridges, using the 7 and 3 as abutments
D-A resin-bonded bridge, using the 7 and 5 as abutments
E-A conventional fixed-moveable bridge using the 7 and 5 as abutments
-----------------------------------------------------------------------------------------
418 Epidemiological studies have shown that dental decay is normally greater amongst
northerners and those in socially deprived circumstances. Which of the following would be most
effective in reducing caries in a high risk population?
A-Brushing with fluoride toothpaste
B-Publicity campaign
C-Fluoridation of the water supply
D-Dietary advice via schools
E-Fissure sealant provision
--------------------------------------------------------------------------------------------
419 A 58 year old male presents at your surgery complaining of a sharp pain of no more than 30
minutes duration arising from his upper left molar region. The pain is brought on by cold stimuli
but persists after-the stimulus is removed. It does not seem to occur spontaneously. He has
tried taking paracetemol and this does temporarily stop the pain from recurring. The upper left 6'
reacted to a lower current on electronic pulp testing than the upper right 6, upper left ~ or the
lower left molars.
What is the most likely cause of the patient's pain? -
A-Acute/reversible pulpitis
B-Dentine sensitivity
C-Chronic/irreversible pulpitis
D-Periapical periodontitis
E-Trigeminal neuralgia
------------------------------------------------------------------------------------------
420 A 60 year old patient attends your surgery complaining of a sore mouth. He has Type II
diabetes well controlled by diet and metformin. On examination white patches which cannot be
removed are present on his buccal mucosa:
A-What is the most likely diagnosis?
B-Frictional keratosis
C-Leukoplakia
D-Lichen Planus
E-White sponge naevus
D-Candidosis
----------------------------------------------------------------------------------------
421 A 60 year old female attends your surgery complaining of soreness affecting her gingivae.
No other area of her oral mucosa is affected but she has noticed an itchy rash on the flexor
surface of her forearms. She is fit and well and is not taking any medication. Scattered
purple/red papules each about 4mm in greatest dimension are present on the flexor surface of
her forearms and on intraoral examination a desquamative gingivitis is present. Based on the
above findings what is your diagnosis?
A-Mucous membrane pemphigoid
B-Lichen planus
C-Pemphigus vulgaris
D-Erythema multiforme
E-Lichenoid drug reaction
----------------------------------------------------------------------------------------
422 An 80 year old patient presents with an ulcer in the floor of the mouth. This has been
present for several months and has not responded to conventional treatment. An incisional
biopsy is taken. Which of the following histological changes in the epithelium confirm a
diagnosis of squamous cell carcinoma?
A-Hyperkeratosis
B-Acanthosis
C-Dysplasia
D-Invasion
E-Discontinuous epithelium
---------------------------------------------------------------------------------------

423 A 35 year old patient complains of swollen gums. This has been present for several years.
What is the commonest cause of this complaint?
A-Cyclosporin therapy
B-Vitamin C deficiency
C-Chronic Gingivitis
D-Atenolol therapy
E- Pregnancy
--------------------------------------------------------------------------------------
424 A 45 year old patient attends the dental clinic complaining of a clicking jaw. Examination
reveals a reproducible click of the right TMJ when opening wide. Upon asking the patient to
open wide, close with incisors edge-to-edge and then open and close to this position, the click is
absent. From the options below, which one is the most likely diagnosis?
A-Myofascial pain
B-Disc displacement with reduction
C-Disc displacement without reduction
D-TMJ osteoarthritis
E-Arthralgia
---------------------------------------------------------------------------------------
425 Cigarette smoking is considered to be the most important factor next to microbial plaque in
periodontal disease progression. Which of the following is the most important factor in the
disease progression in smokers?
A-Smokers have drier mouths than non-smokers .,
B-Smokers have poorer oral hygiene than non-smokers.
C-Nicotine will impair the chemotactic and phagocytic properties of PMNs.
D-The gingival blood flow is reduced in smokers.
E-Smokers alter the oral environment encouraging the growth of anaerobic bacteria
----------------------------------------------------------------------------------------
426 An adult patient attends your practice complaining of pain and swelling associated with a
previously restored upper first premolar tooth. The pain has been present for a number of days
and is no longer responding to analgesics. His dentition is otherwise well maintained and his
periodontal health is good.
What is the most appropriate approach to treatment?
A-Antibiotics and analgesics.
B-Extract the tooth
C-Carry out a pulpotomy. Temporary dressing.
D-Carry out a pulpectomy. Temporary dressing
E- Establish open drainage
-----------------------------------------------------------------------------------
427 A patient reports that his post crown has fallen out. This crown had been present for many
years. You note that there appears to be a hairline vertical fracture of the root. The tooth is
symptomless.
What is the most sensible approach to treatment?
A-Replace the post crown using a resin-reinforced glass ionomer material
B-Replace the post crown using a polycarboxylate cement
C-Replace the post crown using a dentine bonding agent and a resin- reinforced glass ionomer
material
D-Replace the post crown using a resin composite luting agent
E-Arrange to extract the tooth
---------------------------------------------------------------------------------------
428 A patient says that he does not like the appearance of his previously root filled upper
central incisor tooth. His dentition is
otherwise well maintained and his periodontal health is good. The tooth appears to be darker
than the adjacent teeth.
What is the most appropriate approach to treatment?
A-Provision of a post crown
B-Provision of an all ceramic crown
C-Provision of a metal bonded to ceramic crown
D-Carry out a non vital bleaching procedure
E-Provision of a porcelain veneer
------------------------------------------------------------------------------------------
429 A 20 year old patient attends your surgery for the first time. You suspect that he may have
proximal caries as he has a frequent sugar intake. Which of the following is the most accurate
method of diagnosing proximal caries in a lower molar tooth in this young adult?
A. Clinical history
B-Periapical radiograph
C-Bitewing radiograph X
D-Digital image
E-Electronic resistance measurements
-------------------------------------------------------------------------------------------
430 A 25 year old male attends for the first time complaining of sensitivity of a number of teeth.
On examination, the occlusal surfaces of all the teeth are worn with obvious wear facets on the
canines and premolars. Posterior amalgam restorations are proud of the surrounding tooth.
What would be the first stage management?
A-Take impressions for study models
B-Prescribe fluoride mouth rinse
C-Replace the amalgam restorations
D-Dietary analysis
E-Placement of resin sealant to sensitive teeth
--------------------------------------------------------------------------------------
431 A 35 year old male patient who admits to grinding his teeth at night has a number of wedge-
shaped cervical (Class V) lesions on his upper premolar teeth. These are causing some
sensitivity and are approximately 3mm deep. What is the correct management option?
A-Provide tooth brushing instruction and fluoride
B-Restore the lesions with compomer
C-Restore the lesions with micro-filled composite
D-Restore the lesions with a hybrid composite
E. Restore the lesions with conventional glass-ionomer
----------------------------------------------------------------------------------------
432 A patient attends with pain of four days duration in a carious upper molar tooth. The pain is
constant and is not relieved by paracetemol. Sleep has been disturbed by the pain. The tooth is
tender to percussion and gives a positive response to Ethyl Chloride. What is the most likely
diagnosis? .,
A-Pericoronitis
B-Apical periodontitis
C-Marginal periodontitis
D-Reversible pulpitis
E-Irreversible pulpitis
-----------------------------------------------------------------------------------------
433 You are trying in a partial chrome denture framework which fails to seat properlv. It fits the
master cast. What is the most likely cause of this problem?
A-Insufficient expansion of the investment material
B-Distortion of the impression
C-Contraction of the metal framework during casting
D-Failure to block out unwanted undercuts
E-Complex denture design
---------------------------------------------------------------------------------------
434 Bitewing radiography is the main special text used to help in diagnosis of proximal caries.
The performance (accuracy) of a diagnostic test like bitewing radiography can be expressed in
terms of sensitivity and specificity.
Which of the following is a reasonable summary of the diagnostic accuracy of bitewing
radiography for proximal caries diagnosis?
A-Moderate sensitivity and low specificity
B-Moderate sensitivity and moderate specificity
C-Moderate sensitivity and high specificity
D. High sensitivity and moderate specificity
E. High sensitivity and high specificity
------------------------------------------------------------------------------------------
435 You are interested in finding out what the risk indicators are for a rare form of oral cancer
and decide to undertake a study to examine this. What type of study would be the most
appropriate for addressing this issue?
A-Cohort
B-Prevalence study
C-Clinical trial
D-Case-control study
-----------------------------------------------------------------------------------------
436 You take a panoramic radiograph of a patient and discover a well-defined, corticated
radiolucent area below the inferior
dental canal just anterior to the mandibular angle. What is the most likely diagnosis?
A-Radicular cyst
B-Stafne bone cavity
C-Metastatic carcinoma of the breast
D-Adenomatoid odontogenic tumour
E-Complex odontome
---------------------------------------------------------------------------------------------
437 You take a panoramic radiograph out of a patient's records but you find that the film has a
low density and poor contrast.
Which of the following errors could lead to low density and poor contrast?
A-Too long a development time
B-Developer temperature too high
C-Developer temperature too low
D-Inadequate fixation

438 You want to evaluate the effectiveness of using tetracycline as an adjunct to scaling and
root planing for the treatment of
chronic periodontitis. What type of primary study design would be most appropriate for
addressing this topic?
A-Cohort study
B-Non-randomised controlled trial
C-Randomised controlled trial
D-Case-control study
E. Case-series
----------------------------------------------------------------------------------------
439 Radiation protection of patients is partly dependent upon equipment factors (x- ray set and
film or digital system). The
different factors that can be changed vary in financial cost to the dentist and in their
effectiveness in cutting x-ray dose.
Which of the following provides the most cost-effective means of minimizing patient radiation
dose in dental intraoral radiography?
A-Constant potential ('DC) x-ray set
B-Rare earth filtration
C-Lead apron
D-D speed film
E-F speed film
-----------------------------------------------------------------------------------------
440 A 42 year oldman presented with a firm fixed swelling in the right preauricular region which
had been enlarging over the last
month. He had also developed a right sided facial palsy.
What is the most likely diagnosis?
A-Adenoid cystic carcinoma
B-Squamous carcinoma
C-Sebaceous cyst
D-Branchial cyst
E-Pleomophic adenoma
---------------------------------------------------------------------------------------
441 A 14 year old patient attends with a decayed and extensive hypoplastic LL7: He is a very
irregular attender with poor oral
health habits. A radiograph shows the presence of an unerupted LL8 and the LL6 is sound.
What would be the most appropriate long-term treatment for this tooth?
A-Amalgam restoration
B-Antibiotics
C-Extraction
D-Root canal therapy
E-Sedative dressing
------------------------------------------------------------------------------------------
442 A 7 year old boy has previously had all primary molars restored and a pulpotomy on upper
right E. He has an early mixed
dentition with lower lateral incisors erupting. There is a midline diastema of 2 mm. The upper
right E has become symptomatic and
requires extraction.The most likely long term effect of the extraction on the occlusion is:
A-Early eruption of the second premolar.
B-Loss of upper central line.
C-No significant effect.
D-Overeruption of the lower right teeth.
-----------------------------------------------------------------------------------------
E. Potential crowding in the upper right quadrant
443 Which of the following is the ideal treatment for a degree II furcation involvement of a
mandibular molar?
A-Tunnel preparation
B-Root resection
C-Furcation plasty
D-Extraction
E-Guided Tissue Regeneration
-----------------------------------------------------------------------------------------
444 You examine a patlent and find BPE code 4 in all sextants. Radiographs show generalised
horizontal bone loss with a minimum
of 50% of bone support remaining on all teeth.
Which of the following is the most important factor when considering the prognosis for the teeth?
A-Age of the patient
B-The Oral Hygiene Status
C-Bleeding on Probing Score
D-Mobility
E. Gingival Recession
-----------------------------------------------------------------------------------------
445 A 30 yr-old patient attends complaining of occasional pain from the lower left quadrant.
Clinical examination reveals an
extensively restored dentition with generally good oral hygiene. There is no significant
periodontal pocketing other than an
isolated defect in the region of the furcation of lower left first molar which is non-mobile. The
gingival tissue in this area appears
erythematous and slightly hyperplastic with a purulent exudate on probing.
From the list below, which is the most appropriate 'next step?
A. Obtain a radiograph
B-Biopsy the gingival tissue
C-Remove the restoration
D-Vitality testing
E-Prescribe antibiotics
----------------------------------------------------------------------------------------
446 A 40 yr old patient had root-canal treatment to his upper first molar. This was performed 6
months ago using contemporary
techniques under rubber dam and was crowned after completion of treatment. He attends
complaining of continued discomfort
from this tooth. Radiographic examination shows each of the three roots to be obturated with a
well-condensed filling to the full
working length though there is no evidence of in-fill of the periapical lesion when compared to
the pre-op view. There is crestal
bone loss and no furcal involvement. What is the most likely cause of the continued problem?
A-Extra-radicular infection
B-Contamination of canal(s) with E.faecalis
C-Uninstrumented canal
D-Vertical root fracture
E. Perio-endo problem
------------------------------------------------------------------------------------------
447 A mother is concerned that her child's adult upper front teeth have not erupted and asks
your advice.
What is the usual age in years for the upper permanent central incisors to erupt?
A-5 years.
B-6 years.
C-7 years
D-8 years.
E-9 years.
---------------------------------------------------------------------------------------
448 You decide to refer an eight year old child to the oral surgery department in your local
hospital for extractions under a general
anaesthetic. What key reason for asking for a general anaesthetic would you Rut in the referral
letter?
A-Parents request GA.
B-Failed to complete treatment under inhalation sedation
C-Child would not accept local analgesia
D-Parents think hospital more convenient.
E-Not one of my regular patients.
-------------------------------------------------------------------------------------
449 An eight-year-old boy presents with pain of three days duration that has kept him awake.
On examination you see a grossly
carious lower left 6 and some associated buccal swelling.
Which of the following is the most appropriate to give immediate relief of his pain?
A-Extract the LL6.
B-Gently excavate the.cartes and obtain drainage.
C-Give antibiotics.
D-Incise any swelling
E-Refer for-general anaesthetic
---------------------------------------------------------------------------------------
450 A 10 year old girl tripped and-knocked out her upper incisor 10 minutes ago. She is holding
it in her hand.
What is your most effective immediate treatment?
A-Accept the tooth is lost and do nothing.
B-Refer to specialist.
C-Re-plant immediately.
D-Root fill, clean and re-plant.
E-Sterilise root in alcohol and re-implant.
--------------------------------------------------------------------------------------
451 You are treating a patient who has a long history of recurrent episodes of oral candidosis.
His mouth has recently become sore once again and you want to prescribe some antifungals for
him. You check his medical history and find suffers from atrial
fibrillation and is taking warfarin.
Which of the following drugs would be most appropriate to treat this patient's candidosis?
A-Metronidazole
B-Nystatin
C-Fluconazole
D-Miconazole
E-Amoxycillin
----------------------------------------------------------------------------------------
452 A dental company has claimed that ozone is better than conventional methods for treating
decay in peoples' mouths.
Which of the following would provide the best evidence to support these claims?
A-Systematic review of Randomised Controlled Trials
B-Randomised controlled trial
C-Cohort study
D-Case/control study
------------------------------------------------------------------------------------------
453 A GDP in Manchester randomly selects 5000 patients, aged between 45 and 55 years, who
are free from periodontitis. He
determines that 300 of them are smokers. He follows all 5000 patients for 10 years, by which
time 150 had developed
periodontitis. Of the patients with periodontitis, 60 had previously been identified as smokers
and 90 as non-smokers.
What type of study is this?
A-Case-se ries
B-Prevalence study
C-Clinical trial
D-Case-control study
E-Cohort
-------------------------------------------------------------------------------------------
454 Randomised controlled trials enable us to establish cause and effect.
The main reason for using this study design is to:
A-Ensure that making a Type II error is minimised.
B-Ensure that test and control groups are similar.
C-Ensure that test and control groups are of equal size.
D-Help recruitment of patients into the trial.
E-Provide sufficient power for the study
-----------------------------------------------------------------------------------------.
455 A 53 year old male patient presents with an asymptomatic white patch on the ventral
surface of his tongue. He has smoked 35 cigarettes a day since he was 17. An incisional biopsy
ofthe white patch is likely to show which histological feature:
A-Basal cell liquefaction
B-Hyperkeratosis
C-Saw tooth rete ridges
D-Acantholysis
E. Basal cell hypoplasia
---------------------------------------------------------------------------------------
456 A 46 year old female presents with a slowly enlarging painless firm swelling in the hard
palate to the left of the midline.
The most likely diagnosis is:
A-dental abscess
B-Torus palatinus
C-Osteoma
D-Pleomorphic adenoma
E-Canalicular adenoma
--------------------------------------------------------------------------------------
457 A 29 year old man has a prosthetic (mechanical) aortic valve. He had bacterial endocarditis
five years ago. He now requires
removal of his upper and lower right third molars. Which antibiotic prophylaxis is recommended
prior to the procedure?
A-Amoxicillin and clindamycin
B-Metronidazole and cephalexin
C-Amoxicillin and gentamycin
D-Amoxycillin and vancomycin
E-Erythromycin and vancomycin
-------------------------------------------------------------------------------------------
459 A 24 year old man presents with a loculated cystic radiolucency in the lower third molar
area approximately 3cm in diameter.
There is no tooth associated with the cyst and there is no bucco-lingual expansion of the
mandible. There is no resorption of the
roots of the overlying second molar tooth.
How would this be treated?
A-Marsupialisation
B-Enucleation and ethyl chloride
C-Block resection
D-Enucleation and Carnoy's solution
E-Enucleation and formalin solution
---------------------------------------------------------------------------------------------
460 A 21 year old female presents for the first time to your practice. She is very upset with the
appearance of her upper left
central incisor. On examination you find healthy oral hard and soft tissues and excellent oral
hygiene. On close examination you
can see that the upper left central incisor is slightly greyer than the upper right central incisor
and has a composite restoration
placed palatally. What is the most appropriate form oftreatment given the information you have?
A-Bleaching with carbamide peroxide in custom formed trays of upper and lower arches
B-A bonded crown
C-A composite veneer
D-A porcelain veneer
E-Non-vital bleaching with carbamide peroxide
-------------------------------------------------------------------------------------------
461 EDTA (ethylene diamine tetra-acetate) has useful roles in certain situations in clinical
dentistry.
When would you use EDTA?

A.As a root end filling material


B. As a pulp capping agent
C. As a root canal chelating agent
D.As a mouthwash
E.As a dentine bonding agent
---------------------------------------------------------------------------------------------
463 Glass ionomer cements are used in restoring Class V cavities.
Which of the following constituents are most likely to be present in glass ionomer'cements?
A Alumino-silicate glass and phosphoric acid
B. Phosphoric acid and zinc oxide
C. Polyacrylic acid and zinc oxide
D.Aluminosilicate glass and polyacrylic acid
E.Itaconic acid and zinc oxide
465 A patient presents with a history of a post-crown having fallen out. The post- crown was
originally placed fifteen years ago
and had been successful up until four months ago since when it has come out and been
recemented four times. At recementation
there was no evidence of any caries. The patient had been a regular attender and not needed
any restorative treatment for the last
eight years. Which of the following is the most likely cause for the failure of the crown?
A-The post was to narrow
B-The post was to short.
C. The root canal treatment was failing.
D- vertical root fracture was present.
E-There were excessive occlusal loads on the tooth.
------------------------------------------------------------------------------------------
466 In dental epidemiology, indices are used to measure the oral health of a population. The
DMF index is commonly used to
measure the prevalence and severity of dental caries.
What is the main limitation of this approach?
A-It does not allow statistical analysis.
B-It gives equal weight to decayed, missing and filled teeth.
C- It is difficult to calibrate examiners.
D-It is reversible.
E-There is no gold standard
---------------------------------------------------------------------------------------
467 The biological process by which the architecture and function of the lost tissue is completely
restored".
In Periodontology this is a definition of which of the following
A-New Attachment
B-Regeneration
C- Repair
D-Primary Healing
E-Reattachment
-------------------------------------------------------------------------------------------
468 There has been much debate regarding the effectiveness of fluoride in water for preventing
tooth decay. A systematic review
of the evidence drew conclusions as to the reductions in decay which can be expected.
Fluoridation of public water supplies:
A-Reduces tooth decay by 10%.
B-Reduces tooth decay by 20%.
C-. Reduces tooth decay by 30%.
D-Reduces tooth decay by 40%.
E-Reduces tooth decay by 50%
-----------------------------------------------------------------------------------------
469 An obese 40 year old who takes metformin is seen at the end of a morning clinic. After
administration of local analgesia in the
supine position the patient complains of feeling unwell. She is pale and sweating and is
confused. No pulse was detectable. She
was given oxygen and maintained in the supine with no improvement in condition. What would
be the most appropriate drug to administer next?
A-Glucagon
B-Glucose
C-Adrenaline
D-Hydrocortisone
E-Chlorphenamine
------------------------------------------------------------------------------------------
470 A 12 year old boy is brought in by his mother who is concerned about the appearance of his
central and lateral maxillary
incisors. She says they look like they might be decayed. They are a new family to the practice
having recently moved to the area
from Birmingham. On examination you notice a generalised whitening of the incisors which
seems to worse at the incisal edges.
After you have dried the teeth you notice that the whiteness appears to be linear running from
mesial to distal. There is a similar
pattern on the lower incisors.
What is the most likely explanation for the presentation of the boy's incisors?
A-Tetracycline staining
B-Early caries
C-Fluorosis
D-Demarcated Enamel Opacities
E. Previous treatment with composites
------------------------------------------------------------------------------------------
471 A 23 year old patient attends complaining of pain in an upper right molar and is keen to
keep the tooth. The pain is typically
sharp in nature, is triggered by cold and persists after removal of the cold stimulus. The tooth is
not tender to percussion; a
radiograph of the upper right first molar shows a large radiolucency extending to the pulp horn
but no peri-radicular changes.
What treatment is most likely needed in this case?
A-Oral hygiene instruction and fluoride application
B-Excavation of caries and placement of a permanent restoration
C-Root-canal treatment
D-Indirect pulp cap and restoration
E-Direct pulp cap and restoration
---------------------------------------------------------------------------------------------
472 A child of 5 years attends with pain from a grossly decayed lower right D which has a
discharging sinus. He is a hemophiliac.
Which of the following is the most appropriate treatment to relieve his pain?
A-Antibiotics
B-Extraction
C-Fluoride application
D-Non-vital pulpotomy
E-Vital pulpotomy
---------------------------------------------------------------------------------------------
473 You are designing a partial denture for a patient with several missing teeth in the maxilla.
The reason for surveying the model prior to designing the denture is to:
A-Measure and mark out hard and soft tissue undercuts on the casts
B-Relate the intended position of the inter-papillary plane of the patient to the casts
C-Establish the position of the post dam
D-Relate the maxillary and mandibular casts
E-Aid setting up the prosthetic teeth prior to trial insertion
----------------------------------------------------------------------------------------
474 The parotid gland is one of the major salivary glands that supply the oral cavity.
Where is the orifice of the duct of the parotid gland located?
A-At the hamular notch
B-In proximity to the incisive papilla
C-On the buccal mucosa near the maxillary second molar
D-Slightly posterior to the mandibular central incisors
E-Distal to the maxillary third molars on the palatal side
------------------------------------------------------------------------------------
475 Any patient receiving treatment under IV sedation must have their blood pressure recorded
as part of their assessment.
What is the maximum blood pressure that is generally regarded as being compatible with safe
sedation in general dental practice?
A.160/95
B.140/95
C-160/90
D-170/100
E.120/80
------------------------------------------------------------------------------------------
476 A 13 year old boy with Down's syndrome attends for the first time. He is cooperative and
has no relevant medical history. He
is caries free, apart from two small occlusal cavities in his lower second primary molars. His
mother requests that these teeth are
restored.
Which of the following is the most appropriate management?
A-Arranging to extract these teeth at a future visit.
B-Arranging to place two simple restorations at a future visit.
C-Reassurance that these teeth can be left until they exfoliate naturally, with no treatment being
necessary.
D-Recommending use of fluoride mouthwash.
E-Take OPG to check on unerupted teeth.
---------------------------------------------------------------------------------------
477 A 3 year old child attends, showing evidence of bottle caries and also decay affecting
primary molars. The child's mother has
heard that some sugars are damaging her child's teeth. She is not sure which foods to avoid.
From the following, which are you most likely to suggest she avoids?
A-Bread
B-Cheese
C-Ground almonds
D-Pure fruit juices
E-Whole fruits and vegetables
------------------------------------------------------------------------------------------
478 An adult patient complains of her prominent upper front teeth and receding chin. She says
she did not get 'braces' when she
was younger because the family had to move area a lot. Her oral care and health is good, and
she has a 9 mm overjet.
Where is the most suitable place to refer her? "
A-A GDP friend who has a special interest in orthodontics and has been trained to use fixed
appliances.
B-A specialist practitioner who uses a lot of functional appliances.
C-A private specialist practitioner, because she's too old to get NHS treatment now.
D-A hospital consultant, as it is likely she will need surgery now to correct her problem
E-The nearest dental hospital, although it is 70 miles away.
--------------------------------------------------------------------------------------------
479 You are meeting a three year old patient for the first time. His mother explains that he had a
unilateral cleft lip and palate
that was repaired in infancy. She has heard that orthodontics will usually be required when he is
older. She asks you what is the
commonest orthodontic problem that occurs with a repaired cleft palate. What.:.is the most
appropriate answer?
A-A contracted maxillary arch
B-An anterior open bite.
C-An elongated maxillary arch.
D-An expanded maxillary arch.
E-An increased overjet
---------------------------------------------------------------------------------------------
480 Chronic periodontitis is a disease of the periodontium initiated and sustained by microbial
plaque.
Which of the following is the main diagnostic feature of the disease?
A-Tooth Mobility
B-Loss of attachment
C-Pain
D-Gingival recession
E-Drifting
--------------------------------------------------------------------------------------------
481 Gingivitis and periodontitis are associated with the loss of normal tissue.
Which of the following is the principal cause of the tissue loss?
A-Bacterial enzymes such as collagenase and hyaluronidase
B-Release of lysosomal enzymes and oxygen free radicals by PMNs and macro phages
C-Cytotoxic metabolic by products such as ammonia, hydrogen sulphide and toxic amides
D-Complement activation
E-Release of bacterial endotoxins and exotoxins.
--------------------------------------------------------------------------------------------
482 Localised gingival recession is recession of the gingiva confined to one surface of the tooth.
Which of the following is the most important factor leading to the development of localised
gingival recession?
A-Habit activity.
B-Class 2 division 2 malocclusion with complete overbite
C-Excessive toothbrushing
D-Bony dehiscence
E-Fenestration

----------------------------------------------------------------------------------------------------------------------

483 A 48 year old woman complains of a sore area on the right buccal mucosa adjacent to a
restored tooth. The lesion has a
lichenoid appearance and this is confirmed histopathologically following a biopsy.
Which of the following restorative materials is most frequently associated with lichenoid
changes.?
A-Gold
B-Amalgam
C-Porcelain
D-composite
E-Glass ionomer cement
-------------------------------------------------------------------------------------------
484 A patient attends your surgery complaining of severe pain, swelling and mobility associated
with a lower first molar tooth in
which there is a broken filling. A periapical radiograph indicates that the tooth has not been root
filled and there is loss of apical
lamina dura associated with the distal root and at the bifurcation. The periodontal bone support
is good. There is no significant
pocketing. What is the likely diagnosis?
A-Chronic periapical periodontitis
B-An acute periodontal-endodontic periodontitis
C-A chronic periodontal-endodontic periodontitis
D-An acute periapical periodontitis
E-Para radicular periodontitis
------------------------------------------------------------------------------------------
485 An upper incisor in a 16 year old patient has suffered trauma and the coronal tissue has
been lost. The tooth has been
endodontically treated. How will the teeth best restored?
A-With a fibre post, direct core and crown.
B-With a direct core and crown.
C-With composite
D-With an indirect post-core from a pre-fabricated pattern and a crown
E-With a custom indirect post-core and crown
----------------------------------------------------------------------------------------
486 A 55 year old female presents at your surgery complaining of a sharp pain of no more than
a few minutes duration arising
from her lower incisors. The pain only occurs when she eats or drinks cold or sweet foods and
only lasts for as long as the stimulus
is present. A previous dentist applied a varnish to the teeth affected which seemed to help. The
teeth indicated by the patient
appear healthy but when you blow air onto them the patient experiences the pain.
What is the most likely cause of the patient's pain?
A-Acute/reversible pulpitis
B-Dentine sensitivity
C-Chronic/irreversible pulpitis
D-Atypical odontalgia
E-Trigeminal neuralgia
-----------------------------------------------------------------------------------------
487 As a newly qualified dentist you are offered a contract in which remuneration is on
capitation basis.
What is the most important and well-recognised feature of this method of remuneration?
A-Tends to encourage under-prescribing
B-Tends to encourage high technical quality of work
C-Tends to encourage high output of procedures
D-Tends to encourage over-prescribing
E-Tends to encourage low technical quality of work
--------------------------------------------------------------------------------------------
488 A 50 year old male patient has a Class III jaw relationship with an anterior open bite. It is
planned to restore his lower right
second molar, which has suffered tooth wear and fracture, with an indirect restoration. This
tooth has approximately 2mm of
coronal height. What would be the most suitable approach to restore this tooth?
A-Provide an adhesively retained gold onlay
B-Provide a conventional full crown
C-Increase the vertical dimension and provide a full crown
D-Surgically crown lengthen and provide a gold crown
E-Provide an adhesively retained ceramic onlay
--------------------------------------------------------------------------------------------
489 Gracey curettes are specially designed for subgingival debridement and are site specific.
Which curette is recommended for use on the lingual surface of a lower second molar?
A.Gracey Curette number 1 and 2.
B.Gracey Currette number 3 and 4
C.Gracey Currette number 7 and 8
D. Gracey Currette number 11 and 12.
E. Gracey Currette number 13 and 14.
------------------------------------------------------------------------------------------
490 Lignocaine (2%) is widely used in dental procedures. It is most often used in combination
with epinephrine (1 in 80,000).
In which one of the following patients is the use of epinephrine containing local analgesia
potentially hazardous?
A-Patient with severe hypertension
B-Patient on tricyclic antidepressants
C-Patient who is an alcoholic
D-Patient on monoamine oxidase inhibitors (MAOls)
E-Patient with Grand Mal epilepsy

--------------------------------------------------------------------------------------------
491 An 80 year old male presents at your surgery complaining of a sharp stabbing pain of no
more than 2-3 minutes duration
arising from his upper left pre-molar region. The pain can be brought on by cold stimuli but also
occurs spontaneously and has
been sufficiently severe to wake the patient from sleep. He has tried taking paracetemol but this
has been of no benefit. On
examination the patient has a heavily restored upper left 4, which is vital to electrical pulp
testing and shows no radiographic
evidence of caries. Blowing cold air onto the tooth produces the pain but the pain also occurs
spontaneously when you are
examining the patient. There is no evidence of a crack or fracture in the tooth itself.
What is the most likely cause of the patient's pain?
A-Acute/reversible pulpitis
B-Dentine sensitivity
C-Chronic/irreversible pulpitis
D-Atypical odontalgia
E-Trigeminal neuralgia
--------------------------------------------------------------------------------------------
492 A patient complains of a lower incisor which has been mobile for several months. The
radiograph indicates a normal level of
bony support although the periodontal space has widened. The apical bone appears normal.
The tooth is tender to pressure.
Which of the following tests and or examinations would be most likely to provide a diagnosis?
A-Masticatory muscle palpation
B-Electric pulp test
C-Occlusal examination
D-Ethyl chloride test
E-Hot gutta percha application
---------------------------------------------------------------------------------------------
493 A nine year old boy presents with a class I occlusion with no crowding or overjet with a
grossly carious upper left 6 which is
not suitable for restoration. The upper left 7 is very near to eruption. Upper right 6, lower right 6
and lower left 6 are sound and
fissure sealed. The oral hygiene is good. What is the most appropriate extraction pattern for this
patient?
A-Extract all four 1st permanent molars
B-Extract upper left 6 and lower left 6
C-Extract upper left 6 and lower right 6
D-Extract upper right 6 and upper left 6
E-Extract upper left 6 only
--------------------------------------------------------------------------------------------
494 A new patient attends your practice. At the initial examination, you carry out a Basic
Periodontal Examination (BPE).
What does a BPE score of 1 signify?
A-Periodontal health
B-Probing depths greater than 3.5mm
C-Presence of overhanging restorations
D-Presence of supragingival calculus
E-Presence of bleeding on probing
---------------------------------------------------------------------------------------------
495 A patient presents with a history of pain in the right pre-auricular region. There is an
intermittent click during opening; when
the click is not present the patient can open to a normal range. On examination the masticatory
muscles, including the lateral
pterygoid, on the RHS side are tender. What is the most likely diagnosis?
Bruxism
A-Myofascial pain
B-Disc Displacement with Reduction
C-Osteoarthrosis
D-Disc Displacement without Reduction
--------------------------------------------------------------------------------------------
496 A 23 year old male presents to your surgery. He lost his upper lateral incisors some 10
years ago in a swimming pool accident.
Since then he has been wearing a 'spoon' denture which he now feels in aesthetically
unacceptable. He has sought an opinion on
dental implants but has been told that he would need bone grafting for this to be successful and
he is not prepared to undergo this.
His dentition is excellent with no restorations and a Class I occlusion. He wants some advice on
what the best treatment might be.
Which option would you put first on your list of possibilities?
A-Two fixed - fixed resin bonded bridges using the central and canine teeth
B-Two cantilever resin bonded bridges from the central incisors and canines.
C-Two conventional fixed - fixed bridges from the canine
D-Conventional cantilever bridges from the canines
E-Cobalt chrome partial denture
-------------------------------------------------------------------------------------------
497 All the films which have been manually processed by an unsupervised new trainee dental
nurse in your practice are extremely
dark. Which of the following errors could lead to a dark film?
Insufficient exposure
A-Films have been left in the developer for too long a period of time
B-Films have been in the fixer for too long a period of time
C-The concentration of the developer is too dilute
D-The concentration of the fixer is too dilute
---------------------------------------------------------------------------------------------
498 Your colleague is having problems as every panoramic film that he produces shows
extremely wide anterior teeth which are
also blurred. The film also invariably has the images of the condyles cut off from the sides of the
film. The most likely cause of this
A-The patient is incorrectly positioned too far forward relative to the image layer
B-The patient is incorrectly positioned with the chin too low
C-The patient is incorrectly positioned too far back relative to the image layer
D- The patient is in a slumped position in the machine
E-The patients is incorrectly positioned with the head tilted in the machine
--------------------------------------------------------------------------------------------
499 When an individual is exposed to radiation a certain amount of radiation is needed before
clinical signs of damage to somatic
cells appear. For these effects to occur a minimum radiation dose has to be exceeded and this
is known as:
A-Background radiation dose
B-Threshold dose
C-Equivalent dose
D-Absorbed dose
E-Effective dose
---------------------------------------------------------------------------------------------
500 Dentists are encouraged to carry out a thorough soft tissue examination and be vigilant for
any lesions that might possibly be malignant or have malignant potential. Currently it is not
recommended that dentists use a mucosal staining method for screening the general population
for oral cancer. What is the most likely reason for this advice?

A-Unnecessary intervention where there is a false positive test result


B-False reassurance where there is a false negative test result
C-Low uptake of screening by those with low risk
D-Low uptake of screening by those with high risk
E-Negative result reinforcing existing bad habits
----------------------------------------------------------------------------------------------------------------------------- --
----------

1) Choose from the options above the period of time which should elapse before the next
radiographic review in the scenarios below. Each option may be used once, more than once, or
not at all.
A. 1 month
B. 3 months
C. 6 months
D. 12 months
E. 24 months
F. 36 months

1. A 13-year-old patient designated a shaving a high caries risk.


2. A 15-year-old patient considered to be at moderate risk of future
caries.
3. A 32-year-old patient still considered as at high risk of future caries.
4. A 9-year-old patient at low caries risk.
5. A 25-year-old patient at moderate risk of future caries.
6. A38-year-old patient who has had a full coverage crown placed.
7. A 27-year-old patient who has had orthograde endodontic treatment to UL6.
8. A7-year-old who has had avital pulpotomy following trauma to UL1.
--------------------------------------------------------------------------------------------

2) For each of the following clinical scenarios identify the most appropriate skin/nail condition
from those provided. Each option may be used once, more than once, or not at all
A. Clubbing
B. Erythematous palms
C. Evidence of widespread scratching
D. Flattened nails (koilonychias)
E. Keratotic striations
F. Pitted nails
G. Purpura
H. 'Target' lesions.

1. A 45 year old woman with known liver disease


2. A 56-year-old psoriasis sufferer.
3. A woman with a hypochromicmicrocytic anaemia.
4. A heavy smoker with haemoptysis.
5. A patient with a history of gallstones presenting with dark urine and
jaundice.

1ST EMQ:

1- C
2- D

3- C

4- D

5- D

6- D

8- C

2ND EMQ:

1- B

2- F

3- D

4- A

5- C

3) EMQ

A. Behaviour Shaping
B. Distraction
C. Enhancing Control
D. Modelling
E. Negative Reinforcement
F. Systematic Desensitization
G. Tell-show-do

Match the best descriptions for the different Child Behavioural Management Techniques:

1) Patient given a degree of control through the use of a stop signal


2) Progress through small clear defined steps towards ideal behaviour
3) Demonstrated via positive outcome on a third party and illustrate rewards for performing
appropriately
4) Repeated contacts to hierarchy of fear-producing stimuli gradually
5) Strengthening pattern of behaviour by removing unpleasant stimulus as long as required
behaviour exhibited

-----------------------------------------------------------------------------------------

4)EMQ

A. Aciclovir
B. Amoxicillin
C. Clindamycin
D. Erythromycin
E. Metronidazole
F. Miconazole
G Nystatin

Match the first choice of antimicrobials that you would consider for the following conditions:

1) Prophylactic antimicrobial for the removal of retained roots where there is a clear history of
dry sockets following exodontias and allergy to Penicllins
2) Reflux of fluids into the nose from the oral cavity after upper molar extraction
3) Primary herpetic gingivostomatitis
4) Oral white plaques which could be dislodged leaving raw areas associated with inhaler
treatment in an asthmatic patient
5) Acutely inflamed bleeding gum with necrosis and loss of interdental papillae with halitosis

1ST EMQ:

1- C

2- A

3- D

4- F

5- E

2ND EMQ:

1- D

2- B

3- A

4- F

5- E

5) EMQ:
A. Anticurvature filing
B. Apical force technique
C. Balanced force technique
D. Lateral force technique
E. Standardised technique
F. Stepback technique
G. Steptdown technique

For each of the following scenarios, choose the most appropriate term from the list above

1. You are preparing the coronal part of a sclerosed mesiobuccal canal on UL6 before taking a
working length radiograph.
2. You have determined on a radiograph that the mesiobuccal canal of an UR7 has a significant
curve to the distal.
3. An UL2 requires RCT, you are preparing the apical part of the canal to a round cross-section.
4. You are preparing the three canals found in LL7, and have already prepared the apical part of
the canals.
5. You have run out of your normal Ni Ti files and need to use hand K files to prepare the very
curved root canals of LL6.
-------------------------------------------------------------------------------------------

6) EMQ:
A. 0
B. 1
C. 2
D. 3
E. 4
F. 5
G. 6
H. 7

For each of the following scenarios, choose the most appropriate IOTN category from the list
above

1-Increased overjet greater than 9mm


2-Minor malocclusions including minor contact point displacements less than 1mm
3-Reverse overjet greater than 1mm but less than 3.5mm with incompetent lips
4-Increased and complete overbite with gingival or palatal trauma
5-Point contact displacements greater than 1mm but less than or equal to 2mm

1ST EMQ:

1- G

2- A

3- E

4- F

5- C

2ND EMQ:
1- F

2- B

3- D

4- E

5- C

7) EMQ:

A) Shingles
B) Recurrent apthous stomatitis
C) Pemphigus
D) Lichen planus
E) Crohns disease
F) Acute necrotising ulcerative gingivitis
G) Erythema multiforme
H) Candidiosis

Match the oral manifestations you may expect to see for the above conditions

1) Cobble stoning of mucosa and mucosal tags


2) Iris or target lesions, crusted lips
3) White striae
4) Multiple oral ulcers lasting 1 week to 1 month
5) Unilateral vesiculating rash
6) Blisters which could be blood filled. Nikolskys sign
7) White plaques which can be wiped off leaving a red base
8) Ulcers at the tips of interdental papillae
-----------------------------------------------------------------------------------------

8) EMQ:

A- 0
B- 1
C- 2
D- 3
E- 4
F- *

For each of the following scenarios choose the most appropriate BPE score from the options
above.

1. The black band of the WHO probe is partly visible and there is 4mm of recession present
2. There is a ledge on an MO composite on UL4, but no bleeding
3. The whole of the black band of the WHO probe is visible, but there is bleeding on gentle
probing
4. Some but not all of the black band of the WHO probe is visible
5. There is bleeding on probing near the buccal exposed furcation of UR6

1ST EMQ:

1- E

2- G

3- D

4- B

5- A

6- C

7- H

8- F

2ND EMQ:

1- D

2- C

3- B

4- D

5- F

9) EMQ:

A. Lateral pterygoid
B. Masseter
C. Medial pterygoid
D. Temporalis

For each of the following descriptions, choose the most appropriate muscle from the options
above. You may use each option once, more then once or not at all.

1. Inserts on the lateral surface of the angle and lower ramus of the mandible
2. Inserts on the pterygoid fovea below the condylar process of the mandible and the intra-
articular cartilage of the tempero-mandibular joint
3. Inserts on the medial and anterior aspect of the coronoid process of the mandible
4. Action: elevates the mandible and posterior fibres retract
5. Action: elevates, protracts and laterally displaces the mandible to the opposite side for
chewing
-----------------------------------------------------------------------------------------

10) EMQ:

A) The Reporting of Injuries, Diseases and Dangerous Occurrences Regulations


B) Management of Health & Safety at Work Regulations
C) Provision and Use of Work Equipment Regulations
D) Control of Substances Hazardous to Health
E) The Personal Protective Equipment at Work Regulations
F) Health & safety (First Aid) Regulations
G) Pressure Systems and Transportable Gases Regulations
H) Electricity at Work Regulations

Match the following statements about autoclaves with the above regulations under which they
fall.

1-Dental nurses require training on how to operate an autoclave


2-A competent person should draw up a scheme of examination for an autoclave
3-Dental practices have to notify the HSE in the event of an explosion of an autoclave
4-Dental practices have to assess the risks to staff from operating an autoclave
5-An autoclave must be tested periodically for electrical safety

1ST EMQ:

1- B

2- A

3- D

4- D

5- C

2ND EMQ:

1- C

2- G

3- A

4- B

5- H
11) EMQ:
A-Radicular Cyst
B-Dermoid Cyst
C-Nasopalatine cyst
D-Odontogenic Keratocyst
E-Aneurysmal bone cyst
F-Solitary bone cyst
G-Dentigerous cyst
H-Branchial cyst

Match the following key histological features with the above cysts.

1-A cyst wall containing prominent neurovascular bundles C


2-A cyst with keratin in the lumen D
3-Aggregates of multinucleate giant cells associated with dilated vascular channels E
4-The presence of sebaceous glands in the cyst wall B
5-A cyst lining composed of ciliated, pseudostratified columnar epithelium with mucous glands H
6-Rushton bodies A
7-A thin cyst lining composed only of fibrous tissue G
8-A palisaded basal epithelial layer D
---------------------------------------------------------------------------------------
12) EMQ

a) Dietary fibre
b) Dietary protein
c) Folic acid
d) Thiamine
e) Vitamin A
f) Vitamin D
g) Vitamin K

Deficiency of which of the above causes the following clinical presentation?

1) Macrocytic anaemia
2) Night blindness
3) Bleeding diathesis
4) Osteomalacia
5) Peripheral neuropathy

1ST EMQ:

1- C
2- D
3- E
4- B
5- H
6- A
7- G
8- D

2ND EMQ

1- C

2- E

3- G

4- F

5- D

13) EMQ:

A-Angina
B-Aortic aneurysm
C-Myocardial infarction
D-Oesophagitis
E-Pericarditis
F-Pneumonia
G-Pulmonary embolism
H-Herpes zoster

For each of the following scenarios, choose the most appropriate diagnosis from above

1) A 60 year old with acute severe chest pain radiating through to the back with a wide
mediastinum on the chest x-ray
2) A 75 year old smoker with central chest pain radiating to the jaw when he walks to the shops.
The pain resolves at rest.
3) A 40 year old woman with right sided chest pain, worse on breathing in. There is associated
coughing and haemoptysis
4) A 53 year old man with sudden onset chest pain, radiating to the left arm, associated with
sweating and nausea. This pain has been present for 2 hours
5) A 76 year old obese man with episodes of low retrosternal burning pain and difficulty in
swallowing

--------------------------------------------------------------------------------------------

14) EMQ:

A. Acute periapical abscess


B. Cracked tooth syndrome
C. Dentine hypersensitivity
D. Irreversable pulpitis
E. Lateral periodontal abscess
F. Reversable pulpitis

For each of the following problems, choose the most likely diagnosis from above

1) Tooth usually mobile with lateral TTP & associated localised or diffuse swelling of the
adjacent periodontium
2) The tooth is usually extruded with apical TTP. It may be associated with a localised or diffuse
swelling. Radiographic changes show widening of the periodontal ligament
3) Sharp pain on biting. Short duration
4) Diagnosis by elimination and using cold or hot to elicit symptoms
5) Exaggerated response to pulp testing. Carious cavity or leaking restoration present
6) Fleeting pain/sensitivity to hot, cold or sweet with immediate onset. Pain is usually sharp and
maybe difficult to locate. Quickly subsidises after the removal of stimulus

1ST EMQ:

1- B

2- A

3- G

4- C

5- D

2ND EMQ:

1- E

2- A

3- B

4- C

5- D

5- F

15) EMQ:

A-Primary failure of eruption


B-Abnormal position of crypt
C-Early loss of deciduous teeth
D-Supernumerary tooth
E-Congenital absence

For each of the following descriptions, choose the most appropriate from the options above.
1-Failure of eruption of lower permanent molar
2-Canine palatally place
3-Crowding in premolar area
4-Failure of eruption of central incisor
5-Absence of showing lateral incisor

--------------------------------------------------------------------------------------------
16) EMQ:

A-Infiltration + Long buccal


B-Inferior alveolar nerve block
C-Mental nerve block+ buccal +lingual
D-Mental nerve block +lingual infiltration
E-Mental Nerve block

For each of the following descriptions, choose the most appropriate from the options above.

1-Small class III filling of 33


2-Filling mandibular first molar
3-Buccal veneer of 34
4-Surgical removal of 41
5-Pulp removal (hyperemic pulp) on 45

1ST EMQ:

1- A

2- B

3- C

4- D

5- E

2ND EMQ(NOT THAT SURE)

1- E

2- B

3- A

4- D

5- C

17) EMQ:
A Minimal preparation adhesive bridge
B Conventional adhesive bridge
C Fixed-fixed bridge
D Minimal preparation fixed fixed
E Simple cantilever
F Spring cantilever

Choose the most appropriate answer from the options listed above:

1- preferred bridge for replacing UR2


2- preferred bridge for replacing UR1
3- preferred bridge for replacing UR3
4- preferred bridge for replacing UR 6
5- preferred bridge for replacing UR4

-------------------------------------------------------------------------------------------
18) EMQ:

A Lidocaine + adrenaline
B Prilocaine+ felypressin
C Bupivacaine
D Benzocaine
E Articaine

Choose the most appropriate answer from the listed options above:

1)Preferred local anesthesia option for a normal fit pt


2)LA with prolonged duration of action
3)Preferred option of la for pt with heart disease
4)Flavored topical anesthesia
5) LA with the most neuropathological manifestation

You might also like